Download as pdf or txt
Download as pdf or txt
You are on page 1of 672

COMPLETE

Review for
USMLE STEP 2
Intro OB/GYN
In this video, we will cover:

-all OB/GYN concepts


for the USMLE Step 2!
-200 questions!
-awesome mnemonics!
Q1 OB/GYN
How much weight should a severely obese
woman gain during pregnancy?

A) 15 lbs
B) 30 lbs
C) 50 lbs
D) As much as she wants
Q1 OB/GYN
How much weight should a severely obese
woman gain during pregnancy?

A) 15 lbs
B) 30 lbs
C) 50 lbs
D) As much as she wants
Q1
How much weight should a severely obese
woman gain during pregnancy?

A) 15 lbs
B) 30 lbs - average!
C) 50 lbs
D) As much as she wants
Q1
How much weight should a severely obese
woman gain during pregnancy?

A) 15 lbs
B) 30 lbs - average!
C) 50 lbs
D) As much as she wants
Q1
How much weight should a severely obese
woman gain during pregnancy?

A) 15 lbs
B) 30 lbs - average!
C) 50 lbs
D) As much as she wants
Q1
How much weight should a severely obese
woman gain during pregnancy?

A) 15 lbs
B) 30 lbs - average!
C) 50 lbs
D) As much as she wants
Q1
Maternal failure to gain
appropriate weight =
associated with IUGR

Maternal excess weight gain =


associated with
diabetes/macrosomia
Commonly Tested Pregnancy Nutrition Facts
Avoid in pregnancy:
-undercooked meat, fish (mercury toxic to
fetal neurodevelopment), & eggs
-unpasteurized dairy products

Fine during pregnancy


-moderate doses of caffiene
-ginger (for nausea, vomiting)
-probiotics
Q2 OB/GYN
Which prenatal diagnostic test is done during
the 24-28 week period?
A) UA and culture
B) Rubella, RPR/VDRL, HBsAg
C) AFP and quad screen
D) Gestational diabetes
E) GBS culture
Q2 OB/GYN
Which prenatal diagnostic test is done during
the 24-28 week period?
A) UA and culture
B) Rubella, RPR/VDRL, HBsAg
C) AFP and quad screen
D) Gestational diabetes
E) GBS culture
Q2 OB/GYN
Which prenatal diagnostic test is done during
the 24-28 week period?
A) UA and culture - initial visit
B) Rubella, RPR/VDRL, HBsAg - initial visit
C) AFP and quad screen - ~15-20 weeks
D) Gestational diabetes - 24-28 weeks
E) GBS culture - 35-37 weeks
Q2 OB/GYN
Which prenatal diagnostic test is done during
the 24-28 week period?
A) UA and culture - initial visit
B) Rubella, RPR/VDRL, HBsAg - initial visit
C) AFP and quad screen - ~15-20 weeks
D) Gestational diabetes - 24-28 weeks
-but screening is recommended earlier if the patient has obesity and
additional risk factors (e.g., PCOS, prior macrosomic infant)!!!
Q3 OB/GYN
Which of the following values normally
DECREASES pregnancy?
A) Cardiac output
B) Heart rate
C) Blood volume
D) Fibrinogen
E) Expiratory reserve volume
Q3 OB/GYN
Which of the following values normally
DECREASES pregnancy?
A) Cardiac output
B) Heart rate
C) Blood volume
D) Fibrinogen
E) Expiratory reserve volume
Q3 OB/GYN

A) Cardiac output - baby needs blood too!


B) Heart rate - same thing as CO!
C) Blood volume - (but BP decreases first 24 weeks)
D) Fibrinogen - to prevent post-partum hemorrhage
E) Expiratory reserve volume - b/c the diaphragm is
squashed by uterus, decreasing RV and thus ERV
Q4 OB/GYN

What does
G4P2113
mean?
Q4 OB/GYN

What does
G4P2113
a woman who has had:
mean?
2 term deliveries, 1 preterm delivery, 1 abortion, and h
children.
Q4 OB/GYN

What does
G4P2113
a woman who has had:
2mean?
term deliveries, 1 preterm delivery, 1 abortion, and ha
children.
TPAL!
Q4 OB/GYN
G = gravida (number of pregnancies)
T = # term births (carried to 37 weeks)
P = # preterm births (pregnancy between 20 weeks - 36.6 week
A = # abortions (losses before 20 weeks)
L = # living children
Example, G4P2113 means a woman has had
4 pregnancies, 2 passed 37 weeks, 1 preterm, 1 lost before 20 weeks, and 3
living children.
Q5 OB/GYN

Throughout the second trimester, ß-hCG


levels (rise/fall):

A) rise
B) fall
Q5 OB/GYN

Throughout the second trimester, ß-hCG


levels (rise/fall):

A) rise
B) fall
Q5 OB/GYN

Throughout the second trimester, ß-hCG


levels (rise/fall):
~10 weeks

A) rise
B) fall
Q5 OB/GYN

A little bit about ß-hCG...

-ß-hCG is produced by the placenta


-ß-hCG can be detected in the serum (more sensitive &
specific) OR the urine (more convenient)
-ß-hCG levels should double every 48 hours during early
pregnancy; this helps detect ectopic pregnancy
Q6 OB/GYN
In Edward syndrome, what
are the results of the
QUAD-screening?

A) all decreased
B) all elevated
Q6 OB/GYN
In Edward syndrome, what
are the results of the
QUAD-screening?

A) all decreased
B) all elevated
Q6 OB/GYN
In Edward syndrome, what
are the results of the
QUAD-screening?

A) all decreased - "18 is still


under-age"
B) all elevated
Q6 OB/GYN
In Edward syndrome, what
are the results of the
QUAD-screening?

A) all decreased - "18 is still


under-age"
-in DS, ↑hCG , ↑inhibin
Q6 OB/GYN
In Edward syndrome, what
are the results of the
QUAD-screening?

A) all decreased - "18 is still


under-age"
-in DS, ↑hCG , ↑inhibin HI!
Q7 OB/GYN
Maternal serum AFP is decreased (i.e.,
below 0.5 MoM) in:

A) spina bifida
B) gastroschisis
C) omphalocele
D) Trisomy 21
Q7 OB/GYN
Maternal serum AFP is decreased (i.e.,
below 0.5 MoM) in:

A) spina bifida
B) gastroschisis
C) omphalocele
D) Trisomy 21
Q7 OB/GYN
Maternal serum AFP is decreased (i.e.,
below 0.5 MoM) in:

A) spina bifida - elevated


B) gastroschisis - elevated
C) omphalocele - elevated
D) Trisomy 21 - ↓/ x AFP production
Q8 OB/GYN
Nuchal translucency is recommended
at:

A) 11-14 weeks
B) 24-28 weeks
Q8 OB/GYN
Nuchal translucency is recommended
at:

A) 11-14 weeks
B) 24-28 weeks
Q8 OB/GYN
Nuchal translucency is recommended
at:

A) 11-14 weeks -
can detect trisomies &
other gene abnormalities
B) 24-28 weeks
Q9 OB/GYN
Which anti-hypertensive can be taken
during pregnancy?

A) ACEIs
B) ARBs
C) Nifedipine
Q9 OB/GYN
Which anti-hypertensive can be taken
during pregnancy?

A) ACEIs
B) ARBs
C) Nifedipine
Q9 OB/GYN
Which anti-hypertensive can be taken
during pregnancy?

A) ACEIs - neonatal renal failure, IUGR


B) ARBs - same
C) Nifedipine -
also labetolol hydralazine, (methyldopa)
Q10 OB/GYN
Which medication can cause Ebstein anomaly?
A) carbamazepine
B) DES
C) phenytoin
D) tetracycline
E) warfarin
F) streptomycin
G) lithium
H) alcohol
I) retinol
Q10 OB/GYN
Which medication can cause Ebstein anomaly?
A) carbamazepine
B) DES
C) phenytoin
D) tetracycline
E) warfarin
F) streptomycin
G) lithium
H) alcohol
I) retinol
Q10 OB/GYN
Which medication can cause Ebstein anomaly?
A) carbamazepine - neural tube defects, microcephaly
B) DES - clear cell adenocarcinoma of the vagina/cervix
C) phenytoin - IUGR, mental retardation, microcephaly
D) tetracycline - permanent yellow/deciduous teeth
E) warfarin - nasal hypoplasia, developmental delay, IUGR
F) streptomycin - hearing loss, CN8 damage
G) lithium - ebstein anomaly, "Einstein had a lisp"
H) alcohol - fetal alcohol syndrome
I) retinol - mental retardation, SAB, thymic agenesis, etc.
Q11 OB/GYN
Hepatomegaly, "Saber shins", "saddle nose", and
"Hutchinson teeth" are seen in...
A) Congenital toxoplasmosis
B) Congenital rubella
C) Congenital CMV
D) Congenital HSV
E) Congenital HIV
F) Congenital syphillis
Q11 OB/GYN
Hepatomegaly, "Saber shins", "saddle nose", and
"Hutchinson teeth" are seen in...
A) Congenital toxoplasmosis
B) Congenital rubella
C) Congenital CMV
D) Congenital HSV
E) Congenital HIV
F) Congenital syphillis
Q11 OB/GYN
Hepatomegaly, "Saber shins", "saddle nose", and
"Hutchinson teeth" are seen in...
A) Congenital toxoplasmosis - "HIC" triad
B) Congenital rubella - blueberry muffin, PDA, etc.
C) Congenital CMV - periventricular calc., rash
D) Congenital HSV - skin/eye/mouth, CNS infection
E) Congenital HIV - asymptomatic -> FTT
F) Congenital syphillis
Q11 OB/GYN
PREVENTION

A) Congenital toxoplasmosis - avoid cats, raw meat


B) Congenital rubella - immunization b4 pregnancy
C) Congenital CMV - (no vaccine)
D) Congenital HSV - C-section if lesions at delivery
E) Congenital HIV - AZT/nevirapine/C-section if ↑VL
F) Congenital syphillis - penicillin if mom is +
Q12 OB/GYN
A 24-year-old pregnant woman presents at 14
weeks with abdominal pain and some bleeding
from the cervix; POC are found in the vaginal vault
on pelvic exam. What is the next step?

A) Reassurance
B) Ultrasound
Q12 OB/GYN
A 24-year-old pregnant woman presents at 14
weeks with abdominal pain and some bleeding
from the cervix; POC are found in the vaginal vault
on pelvic exam. What is the next step?

A) Reassurance
B) Ultrasound
Q12 OB/GYN
A 24-year-old pregnant woman presents at 14
weeks with abdominal pain and some bleeding
from the cervix; POC are found in the vaginal vault
on pelvic exam. What is the next step?

B) Ultrasound - must determine if all the POC have


been expelled (complete abortion) or not
(incomplete abortion - aspiration or D&C
indicated)
Q12 OB/GYN
Q12 OB/GYN
Q13 OB/GYN
The onset of labor to 6cm dilation is considered
which stage of labor?

• First stage (latent)


• First stage (active)
• Second stage
• Third
Q13 OB/GYN
The onset of labor to 6cm dilation is considered
which stage of labor?

• First stage (latent)


• First stage (active)
• Second stage
• Third
Q13 OB/GYN
The onset of labor to 6cm dilation is considered
which stage of labor?

• First stage (latent) - onset → 6cm


• First stage (active) - 6cm → 10cm
• Second stage - 10cm → baby born
• Third - baby born → placenta out
Q13 OB/GYN
The onset of labor to 6cm dilation is considered
which stage of labor?
*LATENT has 6 letters is it! =)*
• First stage (latent) - onset → 6cm
• First stage (active) - 6cm → 10cm
• Second stage - 10cm → baby born
• Third - baby born → placenta out
Q13 OB/GYN

• First stage (latent) - onset → 6cm "Latent to 6;


• First stage (active) - 6cm → 10cm active to 10;
• Second stage - 10cm → baby born 2nd the baby,
• Third - baby born → placenta out 3rd a placenta, not a hen"
Q13 OB/GYN

• First stage (latent) - onset → 6cm (<20 hrs)


• First stage (active) - 6cm → 10cm (4-6 hours)
• Second stage - 10cm → baby born (0.5-3 hours)
• Third - baby born → placenta out (0-0.5 hours)
Q13 OB/GYN

• First stage (latent) - onset → 6cm (<20 hrs)


• First stage (active) - 6cm → 10cm (4-6 hours)
• Second stage - 10cm → baby born (0.5-3 hours)
• Third - baby born → placenta out (0-0.5 hours)
Note: these values are for a nulliparous woman;
values are less for a multiparous woman
Q14 OB/GYN
What is considered normal FHR?

A) 110 bpm - 160 bpm


B) 70 bpm-150 bpm
Q14 OB/GYN
What is considered normal FHR?

A) 110 bpm - 160 bpm


B) 70 bpm-150 bpm
Q14 OB/GYN
What is considered normal FHR?

A) 110 bpm - 160 bpm


B) 70 bpm-150 bpm

> 160 bpm = tachycardia (e.g., maternal fever)


< 110 bpm = bradycardia (e.g., heart disease)
Q15 OB/GYN
No variability is a sign of

A) a healthy fetus
B) fetal acidemia
Q15 OB/GYN
No variability is a sign of

A) a healthy fetus
B) fetal acidemia
Q16 OB/GYN
Early decelerations are a sign of?

A) head compression
B) uteroplacental insufficiency
C) umbilical cord compression
Q16 OB/GYN
Early decelerations are a sign of?

A) head compression
B) uteroplacental insufficiency
C) umbilical cord compression

"The head comes out early head compression


The placenta comes out late placenta insufficiecy
Very liable with cord compression, cord compression
So make no mistake!"
Q16 OB/GYN
Q17 OB/GYN
What is the treatment for hyperemesis
gravidarum?

A) cheeseburgers & French fries


B) doxylamine-pyridoxine & diet changes
Q17 OB/GYN
What is the treatment for hyperemesis
gravidarum?

A) cheeseburgers & French fries


B) doxylamine-pyridoxine & diet changes
Q17 OB/GYN
Hyperemesis gravidarum
-HG is diagnosed clinically; evaluate for electrolyte abnormalities;
make sure to look out for trophoblastic disease or multiple gestation
(risk factors for HG)
-look out for Wernicke; may develop in severe cases [e.g., ataxia,
oculomotor dysfunction] due to vitamin B1 deficiency; diet changes
and doxylamine-pyridoxine initially; if not working, stop d.p. and
give metoclopramide or promethazine; add ondansetron if still
vomiting.
-if dehydrated, administer IV fluids
Q18 OB/GYN
What is the best initial test for gestational
diabetes mellitus?

A) 50-g glucose challenge test


B) 100-g glucose tolerance test
Q18 OB/GYN
What is the best initial test for gestational
diabetes mellitus?

A) 50-g glucose challenge test


B) 100-g glucose tolerance test
Q18 OB/GYN
What is the best initial test for gestational diabetes
mellitus?

A) 50-g glucose challenge test - screening test;


performed at 24-28 weeks; measures venous plasma
glucose after 1 hour; values > 140 mg/dL are abnormal; if
+, confirm with oral 3 hour 100 g glucose tolerance test
B) 100-g glucose tolerance test
Q18 OB/GYN
Gestational Diabetes
-PE: typically asymptomatic; may present with edema, polyhydramnios, or a
large-for-gestational-age infant
-treatment: ADA diet, regular exercise, strict glucose monitoring (4 times a
day); if diet not enough, add insulin to maintain adequate glucose levels
(e.g., fasting < 95); give intrapartum insulin and dextrose to maintain tight
control during delivery
-monitoring: periodic fetal ultrasound and NSTs for fetal growth and
wellbeing; consider induction at 39-40 weeks
-complications: more than 50% go on to develop glucose intolerance and/or
type 2 DM later in life; screen for DM 6-12 weeks postpartum and repeat
testing every 3 years if normal results
Q19 OB/GYN
Gestational hypertension is defined as systolic
BP >140 or diastolic BP > 90 that develops:

A) any time during pregnancy


B) at or after 20 weeks gestation
Q19 OB/GYN
Gestational hypertension is defined as systolic
BP >140 or diastolic BP > 90 that develops:

A) any time during pregnancy


B) at or after 20 weeks gestation
Q19 OB/GYN
Gestational hypertension is defined as systolic
BP >140 or diastolic BP > 90 that develops:

A) any time during pregnancy - if present before


<20 weeks, it's just regular chronic hypertension
B) at or after 20 weeks gestation; this, PLUS
proteinuria = preeclampsia!
Q19 OB/GYN
Actually... preeclampsia = NEW ONSET HYPERTENSION
(i.e., at >20 weeks) plus ANY of the following:

-proteinuria (e.g., >300 mg of protein in 24 hour period)


-low platelets (i.e., below 100,000)
-elevated creatinine (>1.1, or doubling of creatinine)
-liver transaminases double the upper limit
-pulmonary edema
-cerebral/visual symptoms
Q19 OB/GYN
Preeclampsia = NEW ONSET HYPERTENSION (i.e., at >20
weeks) plus ANY of the following:

-proteinuria (e.g., >300 mg of protein in 24 hour period)


-low platelets (i.e., below 100,000)
-elevated creatinine (>1.1, or doubling of creatinine)
-liver transaminases double the upper limit
-pulmonary edema
-cerebral/visual symptoms

NOTE: THE ONLY CURE FOR PREECLAMPSIA IS DELIVERY!!!


At 37 weeks! And if severe, BEFORE 37 weeks!
Q19 OB/GYN
Preeclampsia = NEW ONSET HYPERTENSION (i.e., at >20
weeks) plus ANY of the following:

-proteinuria (e.g., >300 mg of protein in 24 hour period)


-low platelets (i.e., below 100,000)
-elevated creatinine (>1.1, or doubling of creatinine)
-liver transaminases double the upper limit
-pulmonary edema
-cerebral/visual symptoms

HELLP = hemolysis, elevated LFTs, low platelets


(a variant of preeclampsia with poor prognosis)
Q20 OB/GYN
What precautions are taken for a woman with
preeclampsia?

A) intrapartum magnesium drip


B) seizure prophylaxis for 24 hours post-partum
C) both A & B
Q20 OB/GYN
What precautions are taken for a woman with
preeclampsia?

A) intrapartum magnesium drip


B) seizure prophylaxis for 24 hours post-partum
C) both A & B
Q20 OB/GYN
What precautions are taken for a woman with
preeclampsia?

A) intrapartum magnesium drip


B) seizure prophylaxis for 24 hours post-partum
C) both A & B - and make sure to monitor for magnesium
toxicity (e.g., somnolence, areflexia); ↑risk in patients w/
renal insufficiency b/c magnesium is excreted primarily
by kidneys!; treat with IV calcium gluconate!
Q20 OB/GYN
Reminder!
In patients with Myasthenia Gravis,
magnesium sulfate is contraindicated
because it may trigger a myasthenic crisis
(due to inhibition of acetylcholine release
at the neuromuscular junction)!!
In these patients, seizure prophylaxis is
with valproic acid.
Q21 OB/GYN
What is required in eclampsia management?

A) routine monitoring of magnesium blood levels


B) limitation of fluid intake
Q21 OB/GYN
What is required in eclampsia management?

A) routine monitoring of magnesium blood levels


B) limitation of fluid intake
Q21 OB/GYN

A) routine monitoring of magnesium blood levels - not generally


necessary (unless she has impaired renal function); monitor for
clinical magnesium toxicity
B) limitation of fluid intake - also, control BP (e.g., labetolol and/or
hydralazine); continue magnesium; if seizures recur, give IV
diazepam; monitor fetal status!
Q21 OB/GYN
ECLAMPSIA
-again, deliver is the ONLY cure!! initiate
emergency deliver once the patient is stable and
convulsions are controlled (e.g., magnesium,
diazepam if necessary)
-seizure prophylaxis postpartum (just like in
preeclampsia)
Q22 OB/GYN
During the first-trimester urine culture, a
woman is found to have a positive result; the
patient is asymptomatic. What is the next step?

A) reassurance
B) antibiotics
Q22 OB/GYN
During the first-trimester urine culture, a
woman is found to have a positive result; the
patient is asymptomatic. What is the next step?

A) reassurance
B) antibiotics
Q22 OB/GYN
During the first-trimester urine culture, a
woman is found to have a positive result; the
patient is asymptomatic. What is the next step?

A) reassurance
B) antibiotics - e.g., amox.-clav., or cephalexin
Q22 OB/GYN
What is the treatment for UTI in pregnancy?
-same as asymptomatic bacteriuria! antibiotics!

What is the treatment for pyelonephritis in pregnancy?


-hospitalize!
-IV fluids!
-IV third generation cephalosporins!
Q23 OB/GYN
What is the first step in case of a suspected
ectopic pregnancy?

A) transvaginal ultrasound
B) abdominal ultrasound
Q23 OB/GYN
What is the first step in case of a suspected
ectopic pregnancy?

A) transvaginal ultrasound
B) abdominal ultrasound
Q23 OB/GYN
What is the first step in case of a suspected
ectopic pregnancy?

A) transvaginal ultrasound - and serial hCG


levels if ultrasound is non-diagnostic
B) abdominal ultrasound
Q23 OB/GYN

Reminder:
ectopic
pregnancy
most often
occurs in
the ampulla.
Q23 OB/GYN
Treatment for Ectopic Pregnancy
-medical therapy (e.g., metho-
trexate) for small, unruptured cases
-surgical options: salpingectomy or
salpingostomy with evacuation
-note: tubal rupture or
hemoperitoneum are medical
emergencies!
Q24 OB/GYN
What is occurring in this image?

A) placental abruption
B) placenta previa
C) vasa previa
Q24 OB/GYN
What is occurring in this image?

A) placental abruption
B) placenta previa
C) vasa previa
Q24 OB/GYN
What is occurring in this image?

A) placental abruption
B) placenta previa
C) vasa previa - umbilical cord vessels
pass over the internal os;
-if acute bleed, treat with emergency C-section!
-if diagnosed early, steroids at 28-32 weeks; close monitoring;
scheduled C-section before 35 weeks
Q24 OB/GYN
A little bit about placenta previa...
-placental tissue covers cervix
-most patients are asymptomatic
-significant risk of severe painless antepartum
hemorrhage
-recommend pelvic rest & intercourse abstinence
-many resolve spontaneously.
-routine care; repeat ultrasound in 3rd trimester
-asymptomatic -> C-section at 36-37 weeks
Q24 OB/GYN
And remember...
placental accreta is
attachment of the placenta
to the myometrium.

Usually requires
a hysterectomy to prevent
life-threatening maternal
hemorrhage!!
Q25 OB/GYN
Painful, dark vaginal bleeding is generally
associated with:

A) placental abruption
B) placenta previa
C) vasa previa
Q25 OB/GYN
Painful, dark vaginal bleeding is generally
associated with:

A) placental abruption
B) placenta previa
C) vasa previa
Q25 OB/GYN
Painful, dark vaginal bleeding is generally
associated with:

A) placental abruption - bleeding does not


spontaneously cease
B) placenta previa - painless, bright red
bleeding; ceases within 1-2 hours
C) vasa previa - painless bleeding at ROM
Q25 OB/GYN
Painful, dark vaginal bleeding is generally
associated with:

A) placental abruption - bleeding does not


spontaneously cease
B) placenta previa - painless, bright red
bleeding; ceases within 1-2 hours
C) vasa previa - painless bleeding at ROM
Q25 OB/GYN
Painful, dark vaginal bleeding is generally
associated with:

A) placental abruption - bleeding does not


spontaneously cease
Risk factors include: tobocco or cocaine use, hypertension,
and abdominal trauma; all these can cause ischemia and
necrosis to the maternal vessels at the uteroplacental
interface that result in placental detachment
Q26 OB/GYN
What can cause polyhydramnios?

A) maternal DM
B) multiple gestation
C) duodenal atresia
D) TE fistula
E) cystic lung malformations
F) all of the above
Q26 OB/GYN
What can cause polyhydramnios?

A) maternal DM
B) multiple gestation
C) duodenal atresia
D) TE fistula
E) cystic lung malformations
F) all of the above
Q26 OB/GYN
What can cause polyhydramnios?

A) maternal DM
B) multiple gestation
C) duodenal atresia
D) TE fistula
E) cystic lung malformations
F) all of the above - but often idiopathic!
Q26 OB/GYN
Polyhydramnios

-when AFI ≥ 25 ultrasound


-usually asymptomatic
-complications: preterm labor,
fetal malpresentation, cord prolapse
-tx: not generally treated; planned C-section is
based on etiology
Q27 OB/GYN
Renal agenesis can cause:

A) polyhydramnios
B) oligohydramnios
Q27 OB/GYN
Renal agenesis can cause:

A) polyhydramnios
B) oligohydramnios
Q27 OB/GYN
Renal agenesis can cause:

A) polyhydramnios
B) oligohydramnios - if baby isn't urinating,
there won't be adequate amniotic fluid (can
lead to pulmonary hypoplasia and potter's
sequence)
Q27 OB/GYN
Oligohydramnios

-an AFI < 5 on ultrasound


-dx: based on the sum of the deepest
amniotic fluid pocket in all 4 quadrants
-usually asymptomatic
-tx: treat the underlying cause if possible
Q28 OB/GYN
Who should be
given RhoGAM?

A) Rh+ mothers
B) RH- mothers
Q28 OB/GYN
Who should be
given RhoGAM?

A) Rh+ mothers
B) RH- mothers
Q28 OB/GYN
Who should be
given RhoGAM?
Mom-
A) Rh+ mothers Rho-neg?
B) RH- mothers get
Rho-gam!
Q28 OB/GYN
Isoimmunization
-pathophysiology: fetal RBC’s leak into the maternal circulation (w/
first pregnancy) and maternal anti-Rh IgG antibodies form that can
cross the placenta (w/ next pregnancy) and attack the Rh+ fetus leading
to hemolysis of fetal Rh RBC’s (erythroblastosis fetalis); occurs with
Rh- mother and Rh+ fetus
-prevention: if mother is Rh-, and father is Rh+ or unknown, give
RhoGAM at 28 weeks. if the baby is Rh+, give mother Rhogam
postpartum (or after abortion, ectopic, amniocentesis, vaginal bleeding,
placenta previa, or placental abruption); (dosing is based on Kleihauer-
Betke test; inadequate dosing can lead to alloimmunization)
Q28 OB/GYN

And Remember!
Although the standard dose of Rhogam is usually adequate at 28 weeks
gestation, after delivery or procedures, the Kleihauer-Betke test is used
to determine whether a higher dose is needed due to the increased risk
of fetal RBC's entering the maternal circulation (~50% of cases)!!
Q29 OB/GYN
What is going on?
Q29 OB/GYN
What is going on?
Molar
pregnancy!
(hydatidiform mole)
Q29 OB/GYN
What is going on?
Molar
pregnancy!
(hydatidiform mole)
-transvaginal
ultrasound shows a
mass with cystic
grape-like regions
Q29 OB/GYN
Gestational trophoblastic disease
-Benign GTD: complete (46,XX) and incomplete (69,XXY) molar pregnancies
-Malignant GTD: when the molar pregnancy invades ("invasive HM") or
transforms (choriocarcinoma)
-History: first-trimester bleeding, hyperemesis gravidarum, uterine size
greater than dates
-pelvic exam may reveal enlarged ovaries or expulsion of grapelike molar
clusters
-most accurate test: "snow-storm"/grape-like appearance on ultrasound,
with no gestational sac or fetus; labs show ↑↑ßhCG (>100,000)
-Treatment: evacuate uterus with D&C, follow with weekly ßhCG, treat
malignancy with chemo (e.g., MTX or dactinomycin), possibly hysterectomy
Q30 OB/GYN
Brachial plexus injuries upon
delivery (e.g., Horner
syndrome, Erb-Duchenne palsy,
Klumpke palsy):

A) usually require surgery


B) usually resolve
spontaneously within a year
Q30 OB/GYN
Brachial plexus injuries upon
delivery (e.g., Horner
syndrome, Erb-Duchenne palsy,
Klumpke palsy):

A) usually require surgery


B) usually resolve
spontaneously within a year
Q30 OB/GYN
Risk factors for shoulder dystocia (prolonged 2nd stage of labor):
-obesity
-diabetes
-history of macrosomic infant

Maneuvers
-McRoberts maneuver - leg elevation
-apply suprapubic pressure
-woods screw maneuver - enter vagina, rotation
-passing fetal arm - i.e., pulling out the posterior arm
Q31 OB/GYN
A delivering woman has reached 7cm but there has been no
change in dilation in 6 hours.
What should be done?

A) be patient
B) reassurance
C) amniotomy, oxytocin,
C-section if neither works
Q31 OB/GYN
A delivering woman has reached 7cm but there has been no
change in dilation in 6 hours.
What should be done?

A) be patient
B) reassurance
C) amniotomy, oxytocin,
C-section if neither works
Q31 OB/GYN
Failure to Progress in first stage of labor
-failure to have progressive cervical change
-latent phase vs. active phase
-1st baby vs. 2nd baby

Failure to Progress in second stage of labor


-failure of fetus to descend
-1st baby vs. 2nd baby
-with epidural vs. w/o epidural
Q32 OB/GYN
A woman "breaks her bag of water" at 31 weeks.
What is the next step?

A) emergency surgery in all cases


B) antibiotics, corticosteroids
Q32 OB/GYN
A woman "breaks her bag of water" at 31 weeks.
What is the next step?

A) emergency surgery in all cases


B) antibiotics, corticosteroids
Q32 OB/GYN
A woman "breaks her bag of water" at 31 weeks. What is
the next step?

A) emergency surgery in all cases


B) antibiotics, corticosteroids
antibiotics are given to prevent infection and
corticosteroids (beta-/dexa- methasone) to promote
fetal lung maturity; if infection or fetal distress, then
induce labor; if not, at 34 weeks
Q33 OB/GYN
What is the definition of preterm labor?

A) regular uterine contractions at <37 weeks


B) cervical change at <37 weeks
C) BOTH of the above
Q33 OB/GYN
What is the definition of preterm labor?

A) regular uterine contractions at <37 weeks


B) cervical change at <37 weeks
C) BOTH of the above
Q33 OB/GYN
C) BOTH of the above - preterm labor requires
both criteria
Preterm Labor Treatment
-hydration and bed rest
-tocolytic therapy (beta-mimetics, MgSO4, CCB's, prostaglandin
inhibitors) if <34 weeks gestation, unless contraindicated;
-magnesium for cerebral palsy prophylaxis if <32 weeks;
-steriods to accelerate fetal lung maturity
-penicillin for GBS prophylaxis
Q34 OB/GYN
What is the most common type of breech presentation?
Q34 OB/GYN
What is the most common type of breech presentation?
Q34 OB/GYN
What is the most common type of breech presentation?

Complete Frank
= =
Canon- Feet
ball can
touch
head
Q34 OB/GYN

Breech
-up to 75% spontaneously
change to vertex by week 38
-if not, a version may be
attempted by applying directed
pressure to the maternal
abdomen (50% success rate)
-recommend C-section!
Q35 OB/GYN
Which of the following is/are normal changes
after delivery?
A) transient fever and chills
B) lochia
C) contractions
D) urinary retention
E) suprapubic pain
F) all of the above
Q35 OB/GYN
Which of the following is/are normal changes
after delivery?
A) transient fever and chills
B) lochia
C) contractions
D) urinary retention
E) suprapubic pain
F) all of the above
Q35 OB/GYN
Which of the following is/are normal changes
after delivery?
A) transient fever and chills
B) lochia
C) contractions
D) urinary retention - bladder atony, PN injury
E) suprapubic pain - due to PS diastasis
F) all of the above
Q35 OB/GYN
Also, don't forget about coccydynia (tail bone
pain)!!
-caused by coccyx displaced posteriorly
during vaginal delivery (esp. in prolonged
labor)
-pain usually worse with sitting and leaning
back due to ↑weight on coccyx
-dx: exquisite tenderness on palpation of
coccyx with no signs of infection
-most resolve sponstaneously
Q36 OB/GYN
What is the most common cause of postpartum
hemorrhage (i.e., loss of >1000 mL of blood)?
A) genital tract trauma
B) retained placental tissue
C) uterine atony
Q36 OB/GYN
What is the most common cause of postpartum
hemorrhage (i.e., loss of >1000 mL of blood)?
A) genital tract trauma
B) retained placental tissue
C) uterine atony
Q36 OB/GYN

C) uterine atony
diagnosed by palpation, which reveals a soft, enlarged, "boggy"
uterus; bimanual uterine massage usually successful; also
oxytocin, methergine (if not hypertensive), PGF2α, etc.
-uterine ballon (jada ballon is a newer mechanism)
-if bleeding persists, consider uterine/internal iliac artery
ligation, uterine artery embolization, or hysterectomy
Q37 OB/GYN
A woman presents with a "picket-fence" fever curve
a week after delivery. She does not respond to
antibiotics. What is the most likely diagnosis?
Q37 OB/GYN
A woman presents with a "picket-fence" fever curve
a week after delivery. She does not respond to
antibiotics. What is the most likely diagnosis?

Septic pelvic thrombophlebitis!


Q37 OB/GYN
Septic pelvic thrombophlebitis!
-5 days-3 weeks after delivery
-pelvic infection → vein wall infection and intimal damage →
thrombogenesis → microorganism invasion of the clot →
suppuration/liquefaction → septic embolization
-presents with abdominal and back pain, and swinging fevers
-diagnose with blood cultures, CT to look for pelvic abscess
-treat with broad spectrum antibiotics and heparin
anticoagulation for 7-10 days
Q38 OB/GYN
What is the most
common
presenting
symptom in
Sheehan
syndrome?
Q38 OB/GYN
What is the most
common
presenting
symptom in
Sheehan
syndrome?
-failure to
lactate! (caused
by ↓prolactin)
Q38 OB/GYN
What is the best
What is the most What is the most
initial test for
common accurate test for
Sheehan
presenting Sheehan
syndrome?
symptom in syndrome?
-provocative
Sheehan -MRI of the
hormonal testing
syndrome? pituitary and
-failure to hypothalamus (to
lactate! (caused rule out tumor)
by ↓prolactin)
Q39 OB/GYN
How is uterine inversion
treated?

A) laparotomy
B) reassurance
C) punch it back in
Q39 OB/GYN
How is uterine inversion
treated?

A) laparotomy
B) reassurance
C) punch it back in
Q39 OB/GYN
How is uterine inversion
treated?

A) laparotomy
B) reassurance
C) punch it back in -
after tocolytic agent
Q40 OB/GYN
How is mastitis treated?

A) oral antibiotics
B) breastfeeding
C) both A & B
Q40 OB/GYN
How is mastitis treated?

A) oral antibiotics
B) breastfeeding
C) both A & B
Q40 OB/GYN
How is mastitis treated?

A) oral antibiotics
B) breastfeeding
C) both A & B

-PO antibiotics (dicloxacillin, cephalexin,


augmentin); continued breastfeeding or pump to
prevent accumulation of infected material; if no
improvement in 2-3 days, evaluate for abscess
Q41 OB/GYN
In which tanner stage
does pubic hair appear?
Q41 OB/GYN
In which tanner stage
does pubic hair appear?
Q41 OB/GYN
Stage 1
no pubic hair, flat chest
Stage 2
some pubic hair, breast buds
Stage 3
coarsening hair, mound forms
Stage 4
coarse hair sparing thigh, raised areola

Stage 5
coarse hair - thigh, areola flattens; 15> years
Q42 OB/GYN
Estrogen reaches its peak during which day of
the menstrual cycle?

A) 7
B) 14
C) 19
Q42 OB/GYN
Estrogen reaches its peak during which day of
the menstrual cycle?

A) 7
B) 14
C) 19
Q42 OB/GYN
Estrogen reaches its peak during which day of
the menstrual cycle?

A) 7
B) 14
C) 19
Menstruation & Follicular phase
(day 0 → ~13)
-starts with menstruation, and ends with
LH surge/ovulation
-↑GnRH pulse → ↑FSH → ↑estrogen
production

Ovulation (day 14)


-positive feedback to pituitary →LH surge
→rupture of the ovarian follicle and
release of mature ovum →travels to uterus

Luteal phase (day 15 → 28)


-changes from estrogen to progestorone
predominance; corpus luteum produces
progestorone and some estradiol
-as ↓LH, → ↓estrogen/progestorone by the
corpus luteum → ↓endometrial lining
Q43 OB/GYN
How is menopause diagnosed?

A) clinically
B) ↓LH/↓FSH
Q43 OB/GYN
How is menopause diagnosed?

A) clinically
B) ↓LH/↓FSH
Q43 OB/GYN
How is menopause diagnosed?

A) clinically - average age 51; symptoms


include hot flashes, pruritis, vaginal
atrophy/dryness, insomnia, anxiety, poor
concentration, decreased libido
B) ↓LH/↓FSH - these values are elevated, but
these are not even necessary for diagnosis
Q44 OB/GYN
What is the initial treatment for menopause
symptoms?

A) HRT
B) TCA's
Q44 OB/GYN
What is the initial treatment for menopause
symptoms?

A) HRT
B) TCA's
Q44 OB/GYN
What is the initial treatment for menopause
symptoms?

A) HRT - (but most people just deal with it)


B) TCA's - SSRI's if estrogen not tolerated
Q45 OB/GYN
Which contraception method is permanent?
• Implant (progestin-only implant)
• IUD with progestin
• IUD (copper)
• Tubal ligation
• Medroxyprogesterone
• Transdermal patch
• Vaginal Ring
• OCP's
• Progestin-only "minipills"
• Male condoms
• Female condoms
• Spermicide
Q45 OB/GYN
Which contraception method is permanent?
• Implant (progestin-only implant)
• IUD with progestin
• IUD (copper)
• Tubal ligation
• Medroxyprogesterone
• Transdermal patch
• Vaginal Ring
• OCP's
• Progestin-only "minipills"
• Male condoms
• Female condoms
• Spermicide
Q45 OB/GYN
• Implant (progestin-only implant)
• IUD with progestin >99% effective
• IUD (copper)
• Tubal ligation
• Medroxyprogesterone
• Transdermal patch
• Vaginal Ring 90-90% effective
• OCP's
• Progestin-only "minipills"
• Male condoms
• Female condoms ~80% effective
• Spermicide
Q45 OB/GYN
• Implant (progestin-only implant) - ↑cervical mucus viscosity; up to 3 years
• IUD with progestin - ↑mucus, up to 5 years, spotting, acne, perforation
• IUD (copper) - foreign body inflammation; copper kills; heavy bleeding (8%)
• Tubal ligation - permanently effective, rarely ectopic pregnancy
• Medroxyprogesterone - IM injection, works for 3 months; weight gain
• Transdermal patch - combined estrogen/progesterone; thromboembolism
• Vaginal Ring - combined low dose estrogen/progesterone; place in for 3 weeks
• OCP's - inhibit FSH/LH, suppressing ovulation; thicken mucus; daily compliance
• Progestin-only "minipills" - thicken cervical mucus, strict timed compliance
• Male condoms - only one effective against STD's, HIV
• Female condoms - some protection against STD's; fitted by provider
• Spermicide - inhibits sperm motility; 80% effective
Q46 OB/GYN
Which of the following is a contraindication
to estrogen type contraception (e.g., OCPs)?
A) migraine with aura
B) breast cancer
C) abnormal liver function
D) history of stroke or DVT
E) tobacco use & > 35 years of age
F) all of the above
Q46 OB/GYN
Which of the following is a contraindication
to estrogen type contraception (e.g., OCPs)?
A) migraine with aura
B) breast cancer
C) abnormal liver function
D) history of stroke or DVT
E) tobacco use, > 35 years of age
F) all of the above
Q46 OB/GYN
Which of the following is a contraindication
to estrogen type contraception (e.g., OCPs)?
A) migraine with aura - ↑risk of stroke
B) breast cancer
C) abnormal liver function
D) history of stroke or DVT
E) tobacco use & > 35 years of age
F) all of the above
Q47 OB/GYN
What is the most effective emergency
contraceptive method?

A) Morning-after-pill
B) Oral contraceptive taper
C) Progestin only
D) Copper T IUD
Q47 OB/GYN
What is the most effective emergency
contraceptive method?

A) Morning-after-pill
B) Oral contraceptive taper
C) Progestin only
D) Copper T IUD
Q47 OB/GYN
What is the most effective emergency
contraceptive method?

A) Morning-after-pill - safe, VERY effective


B) Oral contraceptive taper
C) Progestin only - 80% effective
D) Copper T IUD - 99% effective; can continue
to use for up to 10 years; but hard to get Dr.!
Q48 OB/GYN
In which etiology of primary amenorrhea are
LH/FSH elevated?

A) Constitutional growth delay


B) Hypogonadotropic hypogonadism
C) Hypergonadotropic hypogonadism
D) Anatomic problem
Q48 OB/GYN
In which etiology of primary amenorrhea are
LH/FSH elevated?

A) Constitutional growth delay


B) Hypogonadotropic hypogonadism
C) Hypergonadotropic hypogonadism
D) Anatomic problem
Q48 OB/GYN

Hypergonadotropic hypogonadism
in women age <40 =
Primary ovarian insufficiency!!
(↑FSH /↑LH)
Q49 OB/GYN
What test is done to distinguish
Androgen insensitivity syndrome & Mullerian agenesis?
Q49 OB/GYN
What test is done to distinguish
Androgen insensitivity syndrome & Mullerian agenesis?

Karyotype analysis!
46, XY 46, XX
Q50 OB/GYN
A 16-year-old girl presents with ↓appetite,
insomnia, and amenorrhea for 3 months.
What is the most likely diagnosis?
Q50 OB/GYN
A 16-year-old girl presents with ↓appetite,
insomnia, and amenorrhea for 3 months.
What is the most likely diagnosis?

Pregnancy!
Q50 OB/GYN
Workup for secondary amenorrhea:
-pregnancy test
-if negative, measure FSH, TSH, and prolactin
• ↑FSH = POI
• ↑TSH = hypothyroidism
• ↑Prolactin = pituitary pathology; get MRI
-initiate progestin challenge test
-signs of hyperglycemia & hypotension: conduct
dexamethasone suppression test
Q51 OB/GYN
What is the treatment for premature ovarian
failure (in a woman <40 years of age)?

A) Gonadotropins
B) (Estrogen + Progesterone) RT
Q51 OB/GYN
What is the treatment for premature ovarian
failure (in a woman <40 years of age)?

A) Gonadotropins
B) (Estrogen + Progesterone) RT
Q51 OB/GYN
What is the treatment for premature ovarian
failure (in a woman <40 years of age)?

A) Gonadotropins - to induce ovulation in


hypothalamic related amenorrhea
B) (Estrogen + Progesterone) RT - to replace
the hormones the ovary is not making
Q52 OB/GYN
How is primary dysmenorrhea diagnosed?

A) genetic testing
B) FSH & LH levels
C) ultrasound
D) pelvic exam
E) exclusion
Q52 OB/GYN
How is primary dysmenorrhea diagnosed?

A) genetic testing
B) FSH & LH levels
C) ultrasound
D) pelvic exam
E) exclusion
Q52 OB/GYN
Primary Dysmenorrhea
-menstrual pain associated with ovulatory cycles
-no pathologic findings; caused by uterine vasoconstriction,
anoxia, and sustained contractions mediated by an excess of
prostaglandin
-presents with low midline pain, often radiating to the back or
inner thights
-cramps during days 1-3 of menstruation, often with nausea,
diarrhea, headache
-tx: NSAIDs!!, topical heat therapy, OCPs, progestin IUD
Q53 OB/GYN
Pain + menorrhagia + enlarged/boggy uterus:

A) endometriosis
B) adenomyosis
Q53 OB/GYN
Pain + menorrhagia + enlarged/boggy uterus:

A) endometriosis
B) adenomyosis
Q53 OB/GYN
Pain + menorrhagia + enlarged/boggy uterus:

A) endometriosis - uterus not enlarged but


painful nodules
B) adenomyosis
Adenomyosis

A-don't GONNER
know !
TREAT WITH
where GNRH
my AGONISTS
nose is!

dysmenorrhea, abnormal uterine bleedig


Q54 OB/GYN
How is endometriosis diagnosed?

A) laparoscopy
B) ultrasound
Q54 OB/GYN
How is endometriosis diagnosed?

A) laparoscopy
B) ultrasound
Q54 OB/GYN
How is endometriosis diagnosed?

A) laparoscopy - requires direct visualization


B) ultrasound
Q55 OB/GYN
What is the best initial treatment for
menorrhagia?
Q55 OB/GYN
What is the best initial treatment for
menorrhagia?

NSAIDs! (reduce prostaglandin levels)


Q55 OB/GYN
What is the best initial treatment for
menorrhagia?

NSAIDs! (reduce prostaglandin levels)


-tranexamic acid can be given for 5 days during menses
-OCPs, progestin, or a progestin IUD
Q56 OB/GYN
Diagnostic criteria for PCOS include (2/3):
polycystic ovaries (via ultrasound), oligo-
and/or anovulation and:
Q56 OB/GYN
Diagnostic criteria for PCOS include (2/3):
polycystic ovaries (via ultrasound), oligo-
and/or anovulation and:

• evidence of hyperandrogenism
Q56 OB/GYN
Diagnostic criteria for PCOS include (2/3):
polycystic ovaries (via ultrasound), oligo-
and/or anovulation and:

• evidence of hyperandrogenism (clinical


and/or biochemical)
Q56 Reproductive System
Q56 OB/GYN
Polycystic Ovary Syndrome
• Common presentation: obesity, menstrual cycle disturbances, infertility,
acne, androgenic alopecia, and hirsutism
• Diagnosis: biochemical testing of hyperandrogenemia (via ↑free testosterone;
rule out tumor or CAH), evaluate metabolic abnormalities (2-hour glucose
tolerance test, fasting lipid and lipoprotein levels); optional tests (not
necessary if both oligomenorrhea and signs of hyperandrogenism):
transvaginal ultrasound to look for more than 11 small follicles forming a
"pearl necklace" sign (66% of women w/ PCOS), gonadotropins ↑LH/FSH ratio
• Treatment: clomiphene (SERM) +/- metformin is first line for ovulatory
stimulation; if not trying to conceive, combined hormonal contraception! or
progestin; (also, diet & weight loss - for obesity, and can help ovulation)
Q57 OB/GYN

The definition of infertility is the inability to


conceive after how many months of normal
sexual activity?
Q57 OB/GYN

The definition of infertility is the inability to


conceive after how many months of normal
sexual activity?

12 months (if <35 years of age)


6 months (if ≥35 years of age)
Q57 OB/GYN
Notes about infertility
• infertility affects 14% of reproductive age women (i.e., 5
million couples in the US)
• 25% male causes, 58% female, 17% unexplained
• begin with female history and PE, then semen analysis
(because it's so easy & noninvasive), and then further
workup of the woman
• primary infertility means NO prior pregnancies;
secondary infertility means at least 1 prior pregnancy
Q58 OB/GYN

A 25-year-old female comes in with a


unilateral 2-cm cyst at the medial labia
majora. What is the next step?

A) No therapy
B) Drainage
Q58 OB/GYN

A 25-year-old female comes in with a


unilateral 2-cm cyst at the medial labia
majora. What is the next step?

A) No therapy
B) Drainage
Q58 OB/GYN

A 25-year-old female comes in with a


unilateral 2-cm cyst at the medial labia
majora. What is the next step?

A) No therapy -
Bartholin cyst (due to obstruction of the bartholin duct); when small, usually
asymptomatic; larger ones are painful; if develops into abscess, extremely painful; no
therapy +/- warm soaks for asymptomatic cysts; aspiration and drainage for absess -
antibiotics unnecessary unless cellulitis or STI present
Q58 OB/GYN

BIG VIOLIN=
BARTHOLIN
Q59 OB/GYN
Abnormal gray-milky discharge, vaginal
pH > 4.5, + whiff test, > 20% clue cells on
wet mount = ?

A) Bacterial vaginosis
B) Trichomonas
C) Candida
Q59 OB/GYN
Abnormal gray-milky discharge, vaginal
pH > 4.5, + whiff test, > 20% clue cells on
wet mount = ?

A) Bacterial vaginosis
B) Trichomonas
C) Candida
Q59
Abnormal gray-milky discharge, vaginal
pH > 4.5, + whiff test, > 20% clue cells on
wet mount = ?

A) Bacterial vaginosis
B) Trichomonas
C) Candida
METRONIDAZOLE OR TOPICAL AZOLE
VAGINAL
METRO/TINID
OR ORAL
CLINDAMYCIN AZOLE; FLUCONAZOLE
Q60 OB/GYN
How is PID diagnosed?

A) Clinically
B) CT scan
Q60 OB/GYN
How is PID diagnosed?

A) Clinically
B) CT scan
Q60 OB/GYN
How is PID diagnosed?
A) Clinically - acute lower abdominal/ pelvic
pain & either:
• uterine,
• adnexal, or
• CM tenderness;
(hCG to rule out pregnancy)
Q60 OB/GYN
A) Clinically - acute lower abdominal or pelvic pain & either:
uterine, adnexal, or CM tenderness; (hCG to rule out pregnancy)
-PID is an infection of the upper genital tract (30% N gonorrhoeae,
30% C trachomatis)
-give antibiotics while awaiting culture results; examine/treat
partner; outpatient regimen A: ceftriaxone and azithromycin (or
doxycycline if allergy to azithromycin)
-Complications of PID: infertility (10% after first episode, 25%
after second episode); Fitz-Hugh-Curtis syndrome - perihepatitis,
RUQ pain, abnormal liver function
Q60 OB/GYN
Acute Causes of Pelvic Pain: A ROPE
• Appendicitis
⚬ RLQ pain
• Ruptured ovarian cyst
⚬ recent strenuous exersize, minor vaginal bleeding
• Ovarian torsion/abscess
⚬ torsion - severe unilateral pain, nausea; abscess - gradual onset of fever, discharge
• PID
⚬ gradual onset of fever, vaginal discharge
• Ectopic pregnancy
⚬ rule out with ßhCG
Q61 OB/GYN
When do fibroids grow in size?

A) pregnancy
B) after menopause
Q61 OB/GYN
When do fibroids grow in size?

A) pregnancy
B) after menopause
Q61 OB/GYN
When do fibroids grow in size?

A) pregnancy - hormone sensitive


B) after menopause - ↓ estrogen;
menopausal patients typically have
spontaneous fibroid regression with
improvement in symptoms
Q61 OB/GYN
Fibroids (Leiomyoma)
-the most common benign neoplasm of the female genital tract
-composed of smooth muscle and connective tissue
-may cause infertility (similar to an IUD!) or menorrhagia
-very low risk of progression to leiomyosarcoma (0.2%)
-Tx: if asymptomatic, no treatment; symptomatic, combined
hormonal contraception, medroxyprogesterone acetate or
danazol to stop bleeding, GnRH analogs (leuprolide or nafarelin)
to ↓size of myomas; consider myomectomy or hysterectomy (if
done having children)
Histology of Leiomyoma shows:
a whorled pattern of smooth muscle
bundles
Q61 OB/GYN
A woman in pregnancy may present with contractions and
with acute onset abdominal pain along with an associated
mass and leukocytosis.
On exams, this is a degenerating uterine leiomyoma!
Q62 OB/GYN
Vaginal bleeding is always present in
endometrial carcinoma. T/F.

A) True
B) False
Q62 OB/GYN
Vaginal bleeding is always present in
endometrial carcinoma. T/F.

A) True
B) False
Q62 OB/GYN
Vaginal bleeding is always present in
endometrial carcinoma. T/F.

A) True
B) False - vaginal bleeding present 80% of
the time; vaginal pain is a later finding
Q63 OB/GYN
Which type of endometrial cancer grows
with unopposed estrogen stimulation?

A) Type I: endometrioid
B) Type 2: serous
Q63 OB/GYN
Which type of endometrial cancer grows
with unopposed estrogen stimulation?

A) Type I: endometrioid
B) Type 2: serous
Q63 OB/GYN
A) Type I: endometrioid
unopposed estrogen stimulation (e.g., obesity, tamoxifen use,
estrogen-only therapy); 75% of EC; precursor - hyperplasia;
diagnosed at ~55 years of age; favorable prognosis
B) Type 2: serous
unrelated to estrogen; the p53 mutation is present in 90% of cases;
25% of EC; no precursor lesion; diagnosed at ~67 years of age; poor
prognosis
Q64 OB/GYN
When does cervical cancer screening begin?

A) age 11
B) age 21
C) age 30
Q64 OB/GYN
When does cervical cancer screening begin?

A) age 11
B) age 21
C) age 30
Q64 OB/GYN
Cervical cancer screening
• <21: no screening
• 21-29: Pap smear every 3 years
• 30-65: Pap smear every 3 years, or
cotesting (Pap + HPV test) every 5 years
• 65: stop screening if prior tests negative
Q65 OB/GYN

Lichen sclerosis:
A) presents with intense pruritis,
dyspareunia, and dyschezia
B) is malignant
Q65 OB/GYN

Lichen sclerosis:
A) presents with intense pruritis,
dyspareunia, and dyschezia
B) is malignant
Q65 OB/GYN

Lichen sclerosis:
A) presents with intense pruritis,
dyspareunia, and dyschezia
B) is malignant - may progress
Lichen sclerosis:
"itchy" "scc"
Q66 Reproductive System

Lichen simplex chronicus:


A) presents with leathery vulvar skin
B) is malignant
Q66 Reproductive System

Lichen simplex chronicus:


A) presents with leathery vulvar skin
B) is malignant
Q66 Reproductive System

Lichen simplex chronicus:


A) presents with leathery vulvar skin
B) is malignant
Q66 Reproductive System

Lichen simplex chronicus:


A) presents with leathery vulvar skin
B) is malignant
Leather?
I "like it
simple"!
Q67 Reproductive System

Vulvar cancer:
A) can result from long-standing
lichen sclerosis
B) is associated with HPV
C) both A & B are true
Q67 Reproductive System

Vulvar cancer:
A) can result from long-standing
lichen sclerosis
B) is associated with HPV
C) both A & B are true
Q67 Reproductive System

Vulvar cancer:
A) can result from long-standing
lichen sclerosis
B) is associated with HPV
C) both A & B are true
Q68 Reproductive System

Extramammary Paget disease is


associated with:
A) an underlying carcinoma
B) pruritis, erythema, and crusting
Q68 Reproductive System

Extramammary Paget disease is


associated with:
A) an underlying carcinoma
B) pruritis, erythema, and crusting
Q68 Reproductive System

Extramammary Paget disease is


associated with:
A) an underlying carcinoma
B) pruritis, erythema, and
crusting
Q68 Reproductive System
Q68 Reproductive System
Q69 Reproductive System

Imperforate hymen, if untreated,


leads to:
A) amenorrhea and abdominal pain
B) menorrhagia
Q69 Reproductive System

Imperforate hymen, if untreated,


leads to:
A) amenorrhea and abdominal pain
B) menorrhagia
Q69 Reproductive System

Imperforate hymen, if untreated,


leads to:
A) amenorrhea
B) menorrhagia
Q70 Reproductive System

Clear cell adenocarcinoma (of the


vagina):
A) arises due to persistence of
squamous epithelium
B) is associated with DES exposure
Q70 Reproductive System

Clear cell adenocarcinoma (of the


vagina):
A) arises due to persistence of
squamous epithelium
B) is associated with DES exposure
Q70 Reproductive System glandular
columnar
epithelium

DON'T TAKE
DES,clear?
ARE WE
Q70 Reproductive System glandular
columnar
epithelium

DON'T TAKE
DES,clear?
ARE WE
Q71 Reproductive System

Sarcoma botryoides:
A) affects older females
B) shows spindle-shaped cells, and is
desmin +
Q71 Reproductive System

Sarcoma botryoides:
A) affects older females
B) shows spindle-shaped cells, and is
desmin +
Q71 Reproductive System

Sarcoma botryoides:
A) affects older females
B) shows spindle-shaped
cells, and is desmin +
Q71 Reproductive System
Q72 Reproductive System

Cervical dysplasia:
A) begins at the basal layer and
extends outward
B) typically presents with pain
Q72 Reproductive System

Cervical dysplasia:
A) begins at the basal layer and
extends outward
B) typically presents with pain
Q72 Reproductive System

Cervical dysplasia:
A) begins at the basal layer and
extends outward
B) typically presents with pain
Q72 Reproductive System
Q72 Reproductive System

EEL-
STICKS
=
E6
Q73 Reproductive System

What histologic finding is associated


with cervical dysplasia?
A) Koiloctytes
B) Giant cells
Q73 Reproductive System

What histologic finding is associated


with cervical dysplasia?
A) Koiloctytes
B) Giant cells
Q73 Reproductive System

What histologic finding is associated


with cervical dysplasia?
A) Koiloctytes
B) Giant cells
"KOILED UP" RAISINS
Q73 Reproductive System

HPV

Condyloma Cervical Dysplasia


Q74 Reproductive System

How does cervical carcinoma lead to


renal failure?
A) tumor cell cytokine release
B) physical blockage of the ureters
Q74 Reproductive System

How does cervical carcinoma lead to


renal failure?
A) tumor cell cytokine release
B) physical blockage of the ureters
Q74 Reproductive System

How does cervical carcinoma lead to


renal failure?
A) tumor cell cytokine release
B) physical blockage of the ureters
Q74 Reproductive System
Q75 Reproductive System

Primary ovarian insufficiency:


A) makes pregnancy impossible
B) is diagnosed when FSH is high and
estrogen is low
Q75 Reproductive System

Primary ovarian insufficiency:


A) makes pregnancy impossible
B) is diagnosed when FSH is high and
estrogen is low
Q75 Reproductive System

Primary ovarian insufficiency:


A) makes pregnancy impossible
B) is diagnosed when FSH is high and
estrogen is low
Q75 Reproductive System
Q76 Reproductive System

Which of the following can cause anovulation?


A) eating disorders
B) obesity
C) polycystic ovarian syndrome
D) thyroid disorders
E) POI
F) athletics
G) Cushing syndrome
H) all of the above
Q76 Reproductive System

Which of the following can cause anovulation?


A) eating disorders
B) obesity
C) polycystic ovarian syndrome
D) thyroid disorders
E) POI
F) athletics
G) Cushing syndrome
H) all of the above
Q76 Reproductive System

Which of the following can cause anovulation?


A) eating disorders - ovaries shrink
B) obesity - increased estrogen by adipose tissue, negative fbk
C) polycystic ovarian syndrome - androgen overproduction
D) thyroid disorders - hormone imbalances
E) POI - ovarian failure
F) athletics - exercise hormonal imbalance, and low body fat
G) Cushing syndrome - elevated androgens from adrenal glands
H) all of the above
Q77 Reproductive System

In patients with exercise-induced


amenorrhea, LH levels will be:
A) elevated
B) reduced
Q77 Reproductive System

In patients with exercise-induced


amenorrhea, LH levels will be:
A) elevated
B) reduced
Q77 Reproductive System

In patients with exercise-induced


amenorrhea, LH levels will be:
A) elevated
B) reduced - i.e., functional hypothalamic
amenorrhea; due to severe caloric restriction,
GnRH secretion is disrupted - leads to reduced
LH, FSH, and estrogen
Q74 Reproductive System

Adnexal torsion is:


A) when the ovary and tube flip
around the infundibulopelvic /ovarian
ligaments
B) is not an emergency
Q74 Reproductive System

Adnexal torsion is:


A) when the ovary and tube flip
around the infundibulopelvic /ovarian
ligaments
B) is not an emergency
Q74 Reproductive System

Adnexal torsion is:


A) when the ovary and tube flip
around the infundibulopelvic /ovarian
ligaments
B) is not an emergency
Q75 Reproductive System

The most common pelvic organ


prolapse is:

A) bladder
B) rectum
C) uterus
Q75 Reproductive System

The most common pelvic organ


prolapse is:

A) bladder - anterior compartment


B) rectum - posterior compartment
C) uterus - apical compartment
Q75 Reproductive System

Pelvic organ prolapse occurs due to


damage to the levator ani muscle complex.

Asymptomatic patients (e.g., no urinary


retention) do not require treatment!!
Q75 Reproductive System
And...
Uterine procidentia is a form of
pelvic organ prolapse in which the
ENTIRE uterus herniates through
the vagina along with the
anterior and posterior vaginal
walls. Tx: surgery or pessary (if
poor surgical candidate)
Q76 Reproductive System

Endometritis is treated with:


A) reassurance
B) clindamycin + gentamicin
Q76 Reproductive System

Endometritis is treated with:


A) reassurance
B) clindamycin + gentamicin
Q77 Reproductive System

Follicular ovarian cysts:


A) are uncommon
B) usually resolve spontaneously
Q77 Reproductive System

Follicular ovarian cysts:


A) are uncommon
B) usually resolve spontaneously
Q77 Reproductive System

Follicular ovarian cysts:


A) are uncommon
B) usually resolve spontaneously
Q77 Reproductive System
Q78 Reproductive System

Which of the following are risk factors for


ovarian tumor?
A) older age, early menarche, late
menopause, endometriosis, BRCA1 mutation
B) previous pregnancy, OCP's, tubal ligation
Q78 Reproductive System

Which of the following are risk factors for


ovarian tumor?
A) older age, early menarche, late
menopause, endometriosis, BRCA1 mutation
B) previous pregnancy, OCP's, tubal ligation
Q78 Reproductive System

Which of the following are risk factors for


ovarian tumor?
A) older age, early menarche, late
menopause, endometriosis, BRCA1 mutation
B) previous pregnancy, OCP's, tubal ligation

MORE OVULATIONS, MORE


OVARIAN CANCER
Q79 Reproductive System

Ovarian germ cell tumors arise from:


A) ovary surface epithelium
B) the egg
C) the stroma & follicle
Q79 Reproductive System

Ovarian germ cell tumors arise from:


A) ovary surface epithelium
B) the egg
C) the stroma & follicle
Q79 Reproductive System

Ovarian germ cell tumors arise from:


A) ovary surface epithelium
B) the egg epithelial tumors

C) the stroma
germ&
cellfollicle
tumors

sex cord stromal tumors


Q80 Reproductive System

Brenner tumors:
A) are urothelial like
B) are usually malignant
Q80 Reproductive System

Brenner tumors:
A) are urothelial like
B) are usually malignant
Q81 Reproductive System

A mature cystic teratoma is:


A) a type of germ cell tumor
B) benign
C) the most common ovarian tumor in young females
D) formed of elements from all 3 germ layers
E) may present with hyperthyroidism
F) all of the above
Q81 Reproductive System

A mature cystic teratoma is:


A) a type of germ cell tumor
B) benign
C) the most common ovarian tumor in young females
D) formed of elements from all 3 germ layers
E) may present with hyperthyroidism
F) all of the above
Q81 Reproductive System
Q82 Reproductive System

Granulosa cell tumor:


A) is the most common malignant sex cord
stromal tumor
B) produces estrogen
C) can cause post-menopausal bleeding
D) reveals on Call-Exner bodies
E) has an elevated inhibin tumor marker
F) all of the above
Q82 Reproductive System

Granulosa cell tumor:


A) is the most common malignant sex cord
stromal tumor
B) produces estrogen
C) can cause post-menopausal bleeding
D) reveals on Call-Exner bodies
E) has an elevated inhibin tumor marker
F) all of the above
Q82 Reproductive System
Q82 Reproductive System
Q83 Reproductive System

Sertoli-Leydig cell tumors (of the ovary):


A) are malignant
B) resemble testicular histology
Q83 Reproductive System

Sertoli-Leydig cell tumors (of the ovary):


A) are malignant
B) resemble testicular histology
Q84 Reproductive System

Fibromas:
A) are malignant
B) can cause Meigs syndrome
Q84 Reproductive System

Fibromas:
A) are malignant
B) can cause Meigs syndrome
Q85 Reproductive System

Which pathogen causes acute


mastitis?
A) S. aureus
B) Listeria
Q85 Reproductive System

Which pathogen causes acute


mastitis?
A) S. aureus
B) Listeria
Q85 Reproductive System
Reproductive System
Reproductive System
Q86 Reproductive System

Fat necrosis of the breast:


A) is malignant
B) reveals as a calcified oil cyst
on biopsy
Q86 Reproductive System

Fat necrosis of the breast:


A) is malignant
B) reveals as a calcified oil cyst
on biopsy
Q86 Reproductive System

Fat necrosis of the breast:


A) is malignant
B) reveals as a calcified oil cyst
on biopsy
Q87 Reproductive System

Fibrocystic changes:
A) occur most commonly in older
women (>50)
B) are usually not a risk for
cancer
Q87 Reproductive System

Fibrocystic changes:
A) occur most commonly in older
women (>50)
B) are usually not a risk for
cancer
Q87 Reproductive System

Fibrocystic changes:
A) occur most commonly in older
women (>50)
B) are usually not a risk for
cancer
Q88 Reproductive System

Intraductal papillomas are


associated with:
A) bloody discharges
B) "leaf-like" lobulations
Q88 Reproductive System

Intraductal papillomas are


associated with:
A) bloody discharges
B) "leaf-like" lobulations
Q88 OB/GYN

Intraductal papillomas are


associated with:
A) bloody discharges
B) "leaf-like" lobulations
Q89 OB/GYN
Paget's disease of the nipple is
associated with an underlying
malignancy:
A) true
B) false
Q89 OB/GYN
Paget's disease of the nipple is
associated with an underlying
malignancy:
A) true
B) false
Q90 OB/GYN
Invasive ductal carcinoma of the
breast:
A) forms a firm, fibrous, "rock-hard"
mass with sharp margins
B) may cause nipple retractions
C) both A & B are true
Q90 OB/GYN
Invasive ductal carcinoma of the
breast:
A) forms a firm, fibrous, "rock-hard"
mass with sharp margins
B) may cause nipple retractions
C) both A & B are true
Q91 OB/GYN
Inflammatory carcinoma of the
breast:
A) appears warm, swollen, and red
B) has a good prognosis
Q91 OB/GYN
Inflammatory carcinoma of the
breast:
A) appears warm, swollen, and red
B) has a good prognosis
Q91 OB/GYN
Inflammatory carcinoma of the
breast:
A) appears warm, swollen, and red
B) has a good prognosis
Q92 OB/GYN
ER and PR expressing tumor cells
would respond to:
A) tamoxifen
B) trastuzumab
Q92 OB/GYN
ER and PR expressing tumor cells
would respond to:
A) tamoxifen
B) trastuzumab
Q92 OB/GYN
ER and PR expressing tumor cells
would respond to:
A) tamoxifen
B) trastuzumab
Q93 OB/GYN
Bilateral breast tenderness + fibrocystic changes +
early satiety + and a complex ovarian mass =

A) Granulosa cell tumor


B) Yolk sac tumor
C) Embryonal carcinoma
D) Dysgerminomas
E) Sertoli-Leydig cell tumor
Q93 OB/GYN
Bilateral breast tenderness + fibrocystic changes +
early satiety + and a complex ovarian mass =

A) Granulosa cell tumor= granulosa cells↑estrogen


B) Yolk sac tumor= abdominal pain; no breast pain
C) Embryonal carcinoma= rapid onset pelvic pain
D) Dysgerminomas= no breast pain; painful masses
E) Sertoli-Leydig tumor= virilization; ↓estrogen
Q94 OB/GYN
A 68-year-old woman has severe vulvar itching and
burning for several months. PE shows thin, dry,
white plaque-like vulvar skin with loss of the labia
minora. What is the next step?

A) KOH prep
B) Vulvar punch biopsy
Q94 OB/GYN
A 68-year-old woman has severe vulvar itching and
burning for several months. PE shows thin, dry,
white plaque-like vulvar skin with loss of the labia
minora. What is the next step?

A) KOH prep
B) Vulvar punch biopsy
Q94 OB/GYN
A 68-year-old woman has severe vulvar itching and
burning for several months. PE shows thin, dry,
white plaque-like vulvar skin with loss of the labia
minora. What is the next step?

A) KOH prep
B) Vulvar punch biopsy - confirm diagnosis of
Lichen sclerosis and rule out vulvar cancer
Q95 OB/GYN
What is the best treatment for lichen sclerosus?

A) Topical estrogen
B) Topical steroids
Q95 OB/GYN
What is the best treatment for lichen sclerosus?

A) Topical estrogen
B) Topical steroids
Q95 OB/GYN
What is the best treatment for lichen sclerosus?

A) Topical estrogen
B) Topical steroids - e.g., clobetasol; decreases
chronic inflammation; can also prevent disease
progression to vulvar cancer
Q96 OB/GYN
Hyperandrogenism in pregnancy is commonly due
to benign, bilateral ovarian masses such as theca
lutein cysts and _.

A) Sertoli-Leydig tumors
B) Luteomas
Q96 OB/GYN
Hyperandrogenism in pregnancy is commonly due
to benign, bilateral ovarian masses such as theca
lutein cysts and _.

A) Sertoli-Leydig tumors
B) Luteomas
Q97 OB/GYN
Fever + diffuse abdominal pain worse on one side +
multiloculated complex adnexal mass =

A) Mature cystic teratoma


B) Tuboovarian abscess
Q97 OB/GYN
Fever + diffuse abdominal pain worse on one side +
multiloculated complex adnexal mass =

A) Mature cystic teratoma


B) Tuboovarian abscess
Q98 OB/GYN
A 7-year-old girl with precocious puberty and a
large adnexal mass. What is the diagnosis?

A) Dysgerminoma
B) Granulosa cell tumor
Q98 OB/GYN
A 7-year-old girl with precocious puberty and a
large adnexal mass. What is the diagnosis?

A) Dysgerminoma
B) Granulosa cell tumor
Q99 OB/GYN
An adolescent girl discovers a breast mass with
clinical features consistent with fibroadenoma
(unilateral, firm, mobile, well-circumscribed, upper
outer quadrant). What is the next step?

A) Reassurance and no further testing


B) Observation & reexamination in 6 weeks
Q99 OB/GYN
An adolescent girl discovers a breast mass with
clinical features consistent with fibroadenoma
(unilateral, firm, mobile, well-circumscribed, upper
outer quadrant). What is the next step?

A) Reassurance and no further testing


B) Observation & reexamination in 6 weeks
Q100 OB/GYN
A 10-year-old girl has vulvar itching that has worsened
over the last few months. Exam of the vulva reveals thin,
white skin with excoriations extending to the perianal
area. She has not had her first period yet. What is the
next step?

A) Steroids
B) Fluconazole
C) Biopsy
Q100 OB/GYN
A 10-year-old girl has vulvar itching that has worsened
over the last few months. Exam of the vulva reveals thin,
white skin with excoriations extending to the perianal
area. She has not had her first period yet. What is the
next step?

A) Steroids
B) Fluconazole
C) Biopsy
Q101 OB/GYN
A 27-year-old woman with a previous
abortion comes in at 32 weeks. Fetal tones
are not heart and ultrasound reveals no
cardiac activity. What should you respond
when the patient asks why it happened?

A) I am sorry; this is likely due to an


underlying condition
B) Although we can do further testing, we
may never know the true cause of this
Q101 OB/GYN
A 27-year-old woman with a previous
abortion comes in at 32 weeks. Fetal tones
are not heart and ultrasound reveals no
cardiac activity. What should you respond
when the patient asks why it happened?

A) I am sorry; this is likely due to an


underlying condition
B) Although we can do further testing, we
may never know the true cause for this
Q101 OB/GYN

the cause of intrauterine


fetal demise (i.e., fetal
death ≥ 20 weeks) is most
often unknown!!
Q102 OB/GYN
A 33-year-old woman who delivered 6 weeks ago, with a
complication of severe post partum bleeding now
complains of fatigue, poor appetite, and poor lactation.
She has also lost 20 lbs. relative to her pre-prepregnancy
weight. What is the cause?

A) Postpartum depression
B) Pituitary necrosis
Q102 OB/GYN
A 33-year-old woman who delivered 6 weeks ago, with a
complication of severe post partum bleeding now
complains of fatigue, poor appetite, and poor lactation.
She has also lost 20 lbs. relative to her pre-prepregnancy
weight. What is the cause?

A) Postpartum depression
B) Pituitary necrosis
Q102 OB/GYN

Sheehan syndrome (a life-threatening complication of


postpartum hemorrhage) typically presents with
lactation failure (due to prolactin deficiency) as well as
hypotension and anorexia (due to secondary adrenal
insufficiency due to impaired ACTH secretion)
Q103 OB/GYN
A 27-year-old woman has severe
dyspareunia and dysmenorrhea.
Speculum exam is normal. What is the
best treatment?

A) Vaginal dilators
B) Combination oral contraceptives
Q103 OB/GYN
A 27-year-old woman has severe
dyspareunia and dysmenorrhea.
Speculum exam is normal. What is the
best treatment?

A) Vaginal dilators
B) Combination oral contraceptives
Q103 OB/GYN
A 27-year-old woman has severe dyspareunia
and dysmenorrhea. Speculum exam is normal.
What is the best treatment?

A) Vaginal dilators
B) Combination oral contraceptives -
combination oral contraceptives (and NSAIDS)
for endometriosis (treat inflammation and
suppress ovarian stimulation of endometriosis)
Q103 OB/GYN

Note:
If endometriosis doesn't improve with
medication, surgical evaluation via laparoscopy
is required for definitive diagnosis (e.g.,
biopsy) and resection of the implants!
Q104 OB/GYN
A 31-year-old woman at 30 weeks pregnant was struck in the
abdomen in a car accident. Her pants are soaked with blood, and
her abdomen is in pain. Blood pressure = 89/57 and her
extremities are cool, pulse is 140/min. Uterine contractions
occur every few minutes. IV fluids are started. What is the next
step?

A) Fetal biophysical profile


B) Blood transfusion
Q104 OB/GYN
A 31-year-old woman at 30 weeks pregnant was struck in the
abdomen in a car accident. Her pants are soaked with blood, and
her abdomen is in pain. Blood pressure = 89/57 and her
extremities are cool, pulse is 140/min. Uterine contractions
occur every few minutes. IV fluids are started. What is the next
step?

A) Fetal biophysical profile


B) Blood transfusion
Q104 OB/GYN
B) Blood transfusion
-the car accident likely caused
abruptio placentae
-this requires volume
replacement (switch from
crystalloid to blood product
resuscitation ASAP)
-maternal stabilization is the
priority over fetal evaluation!
Q105 OB/GYN
A woman pregnant with
monochorionic diamniotic
twins is at increased risk of
which complication:

A) Twin-twin transfusion
syndrome
B) Cord entanglement
Q105 OB/GYN
A woman pregnant with
monochorionic diamniotic
twins is at increased risk of
which complication:

A) Twin-twin transfusion
syndrome
B) Cord entanglement
Q105 OB/GYN
monochorionic = 1 placenta
A woman pregnant with -increases risk for TTTS
monochorionic diamniotic where unbalanced AV
twins is at increased risk of anastomoses are present
which complication: between the shared placental
vessels; high pressure from
the arteries from one twin is
A) Twin-twin transfusion shunted to the placental
syndrome veins of the other twin; ↑risk
B) Cord entanglement of mortality in both twins
Q106 OB/GYN
A 26-year-old female bleeds more and more after a
vaginal delivery. BP = 70/39. Hg = 6g/dL, platelets =
70,000. PT/INR and PTT are prolonged. What is the
diagnosis?

A) TTP
B) DIC
Q106 OB/GYN
A 26-year-old female bleeds more and more after a
vaginal delivery. BP = 70/39. Hg = 6g/dL, platelets =
70,000. PT/INR and PTT are prolonged. What is the
diagnosis?

A) TTP
B) DIC
Q106 OB/GYN
A 26-year-old female bleeds more and more after a
vaginal delivery. BP = 70/39. Hg = 6g/dL, platelets =
70,000. PT/INR and PTT are prolonged. What is the
diagnosis?

A) TTP
B) DIC - postpartum hemorrhage is a major cause of DIC
due to large volume of bleeding (tissue factor is
released which leads to ↑↑activated of the CC →
↑thrombi → platelet consumption → ↑PT/PTT)
Q106 OB/GYN
A 26-year-old female bleeds more and more after a
Tx:
vaginal delivery. BP = 70/39. Hg = 6g/dL, platelets =
70,000. PT/INR and PTT are prolonged. What is the emergency
diagnosis? supportive
care and
A) TTP
resus-
B) DIC - postpartum hemorrhage is a major cause of DIC
due to large volume of bleeding (tissue factor is citation
released which leads to ↑↑activated of the CC → with blood
↑thrombi → platelet consumption → ↑PT/PTT) products!
Q107 OB/GYN
A 33-year-old woman presents 6 weeks after
labor with fatigue and poor sleep. She also has
an unsteady gait and increased knee reflexes.
What is the next step?

A) Reassurance
B) MRI
Q107 OB/GYN
A 33-year-old woman presents 6 weeks after
labor with fatigue and poor sleep. She also has
an unsteady gait and increased knee reflexes.
What is the next step?

A) Reassurance
B) MRI
Q107 OB/GYN
A) Reassurance
B) MRI
-fatigue and poor sleep are common postpartum; unsteady
gait and hyperreflexia are not! Concerning for a first
presentation of MS! (fatigue is often the first symptom);
-although pregnancy is protective for MS (due to
immunosuppressive effects of pregnancy) there is ↑risk of
both initial presentation and relapses during the early
postpartum period.
Q108 OB/GYN
A 22-year-old female comes in complaining of excess
facial hair, acne, and irregular menstrual cycles. She
has normal external female genitalia but serum 17-
hydroxyprogestorone and DHEAS are elevated. What
is the diagnosis?

A) Non-classic CAH
B) Polycystic ovary syndrome
Q108 OB/GYN
A 22-year-old female comes in complaining of excess
facial hair, acne, and irregular menstrual cycles. She
has normal external female genitalia but serum 17-
hydroxyprogestorone and DHEAS are elevated. What
is the diagnosis?

A) Non-classic CAH
B) Polycystic ovary syndrome
Q108 OB/GYN

A) Non-classic CAH - ↑ 17-hydroxyprogestorone,


which shifts toward adrenal androgen synthesis →
hyperandrogenism; usually present in late
teens/early adulthood with acne and hirsutism;
B) Polycystic ovary syndrome - hyperandrogenism
but no↑ 17-hydroxyprogestorone
Q109 OB/GYN
A 32-year-old woman at 41 weeks gestation comes in to her
OB/GYN. She complains of lack of movement. Fetal testing shows
late decelerations and the single deepest pocket of amniotic
fluid is 1cm. BPP = 4. What is going on?

A) Fetal head compression


B) Strangling umbilical cord
C) Uteroplacental insufficiency
Q109 OB/GYN
A 32-year-old woman at 41 weeks gestation comes in to her
OB/GYN. She complains of lack of movement. Fetal testing shows
late decelerations and the single deepest pocket of amniotic
fluid is 1cm. BPP = 4. What is going on?

A) Fetal head compression


B) Strangling umbilical cord
C) Uteroplacental insufficiency
Q109 OB/GYN
A 32-year-old woman at 41 weeks gestation comes in to her
OB/GYN. She complains of lack of movement. Fetal testing shows
late decelerations and the single deepest pocket of amniotic
fluid is 1cm. BPP = 4. What is going on?

A) Fetal head compression


B) Strangling umbilical cord
C) Uteroplacental insufficiency - "the placenta comes out late"
Q109 OB/GYN

C) Uteroplacental insufficiency
-also, pregnancies at ≥41 weeks → ↓placental function →↓fetal
perfusion →fetal hypoxemia (late decelerations) →
uteroplacental insufficiency → blood preferentially delivered to
brain, kidneys get dehydrated → oligohydramnios (single deepest
pocket ≥2 cm); UPI can lead to fetal death
Q110 OB/GYN
A 34-year-old woman at 38 weeks has sudden vaginal
bleeding and severe lower abdominal pain. Cervix is
dilated to 3 cm. Contractions occur every 2 minutes.
What is a complication of her state?

A) Hemorrhage
B) Retained placenta
C) Uterine rupture
Q110 OB/GYN
A 34-year-old woman at 38 weeks has sudden vaginal
bleeding and severe lower abdominal pain. Cervix is
dilated to 3 cm. Contractions occur every 2 minutes.
What is a complication of her state?

A) Hemorrhage
B) Retained placenta
C) Uterine rupture
Q110 OB/GYN
A) Hemorrhage
-severe abdominal pain + bleeding at 38 weeks = abruptio
placentae (bleeding caused by placental detachment;
accumulation of blood increases intrauterine pressure →
abdominal and/or back pain); some cases are self-
limited, but complications include hemorrhage,
hypovoemic shock and DIC
-preeclampsia and smoking are risk factors
Q111 OB/GYN
A 29-year-old woman at 17 weeks comes in with
polydipsia, nocturia, and polyruia. Her urinalysis
shows low specific gravity. Serum sodium = 140 mEq/L.
What is the diagnosis?

A) Normal pregnancy changes


B) Diabetes insipidus
Q111 OB/GYN
A 29-year-old woman at 17 weeks comes in with
polydipsia, nocturia, and polyruia. Her urinalysis
shows low specific gravity. Serum sodium = 140 mEq/L.
What is the diagnosis?

A) Normal pregnancy changes


B) Diabetes insipidus
Q111 OB/GYN

B) Diabetes insipidus
-low specific gravity urine indicates DILUTE urine. So she has
dilute urine despite drinking a lot and despite normal serum
sodium - that's diabetes insipidus!
-DI is characterized by an inability to concentrate urine (due
to insufficiency of ADH or a lack of response to ADH)
-can first present at pregnancy as placentally produced
enzymes (e.g., vasopressinase) increase ADH breakdown;
may resolve with delivery of the baby!
Q112 OB/GYN
A 32-year-old woman has a postpartum hemorrhage (a week
after delivery). Exam shows no laceration but the cervix is
slightly dilated and there is active bleeding from the ox. She has
no fever. The uterus is small, firm, and nontender. Pt = 11 sec, PTT
= 36 wec, and BT = 14 minutes. What is the cause of the
postpartum hemorrhage?

A) Uterine atony
B) Retained products of conception
C) Von Willebrand disease
Q112 OB/GYN
A 32-year-old woman has a postpartum hemorrhage (a week
after delivery). Exam shows no laceration but the cervix is
slightly dilated and there is active bleeding from the ox. She has
no fever. The uterus is small, firm, and nontender. Pt = 11 sec, PTT
= 36 wec, and BT = 14 minutes. What is the cause of the
postpartum hemorrhage?

A) Uterine atony
B) Retained products of conception
C) Von Willebrand disease
Q112 OB/GYN
A 32-year-old woman has a postpartum hemorrhage (a week
after delivery). Exam shows no laceration but the cervix is
slightly dilated and there is active bleeding from the ox. She has
no fever. The uterus is small, firm, and nontender. Pt = 11 sec, PTT
= 36 wec, and BT = 14 minutes. What is the cause of the
postpartum hemorrhage?

A) Uterine atony - enlarged, soft uterus


B) Retained products of conception - pelvic pain, fever
C) Von Willebrand disease - postpartum hemorrhage, ↑BT
Q113 OB/GYN
What is this fetal tracing a sign of?

A) Fetal head compression


B) Umbilical cord compression
Q113 OB/GYN
What is this fetal tracing a sign of?

A) Fetal head compression


B) Umbilical cord compression
Q113 OB/GYN
What is this fetal tracing a sign of?

A) Fetal head compression


B) Umbilical cord compression

"The head comes out early head compression


The placenta comes out late placenta insufficiecy
Very liable with cord compression,cord compression
So make no mistake!"
Q114 OB/GYN
What is first-line management of recurrent
variable decelerations (i.e., >50% of
contractions)?

A) Maternal repositioning
B) Amnioinfusion
C) Oxytocin
Q114 OB/GYN
What is first-line management of recurrent
variable decelerations (i.e., >50% of
contractions)?

A) Maternal repositioning
B) Amnioinfusion
C) Oxytocin
Q114 OB/GYN

A) Maternal repositioning -
1st line for recurrent variable decelerations (e.g., left lateral, on all-fours;
may reduce cord compression); if that doesn't work, amnioinfusion may be
given; this provides more amniotic fluid which can reduce cord compression

C) Oxytocin
increases contraction strength/frequency → worsens umbilical cord
compression
Q115 OB/GYN
A 33-year-old woman presents with symptomatic
anemia (e.g., syncope) due to heavy menstrual periods.
Bimanual exam reveals a large irregularly shaped
uterus. Pregnancy test is negative. What is the cause?

A) Endometrial glands and stroma in myometrium


B) Proliferation of smooth muscle in the myometrium
Q115 OB/GYN
A 33-year-old woman presents with symptomatic
anemia (e.g., syncope) due to heavy menstrual periods.
Bimanual exam reveals a large irregularly shaped
uterus. Pregnancy test is negative. What is the cause?

A) Endometrial glands and stroma in myometrium


B) Proliferation of smooth muscle in the myometrium
Q115 OB/GYN
A) Endometrial glands and stroma in myometrium -
adenomyosis; typically uniformly enlarged uterus and
diffuse uterine tenderness
B) Proliferation of smooth muscle in the myometrium
-fibroids (leiomyomas); common benign tumors; subserosal
ones especially cause an irregularly shaped enlarged uterus
and bulk symptoms (e.g., pelvic pressure); submucosal ones
cause heavy bleeding because their protrusion into the
endometrial cavity↑endometrial surface area
Q115
Q116 OB/GYN
A 31-year-old woman comes in for first prenatal visit at
9 weeks gestation. Her first pregnancy required
penicillin during labor for GBS. She lives in a home built
in 1980. What screening test is required at this visit?

A) Serum lead level


B) HBV
C) GBS
Q116 OB/GYN
A 31-year-old woman comes in for first prenatal visit at
9 weeks gestation. Her first pregnancy required
penicillin during labor for GBS. She lives in a home built
in 1980. What screening test is required at this visit?

A) Serum lead level


B) HBV
C) GBS
Q116 OB/GYN
A 31-year-old woman comes in for first prenatal visit at
9 weeks gestation. Her first pregnancy required
penicillin during labor for GBS. She lives in a home built
in 1980. What screening test is required at this visit?

A) Serum lead level - homes built before 1978


B) HBV - universal screening for HIV, HBV, syphillis at
first visit
C) GBS - screening much later on (36-38 weeks)
Q117 OB/GYN
A 20-year-old female presents with high fever (103F),
hypotension, tachycardia, and a diffuse red macular
rash on her entire body (including palms and soles). She
uses tampons for heavy bleeding. BP = 79/39. Which
organ is responsible for her symptoms?

A) S. aureus
B) T. pallidum
Q117 OB/GYN
A 20-year-old female presents with high fever (103F),
hypotension, tachycardia, and a diffuse red macular
rash on her entire body (including palms and soles). She
uses tampons for heavy bleeding. BP = 79/39. Which
organ is responsible for her symptoms?

A) S. aureus
B) T. pallidum
Q117 OB/GYN
A 20-year-old female presents with high fever (103F),
hypotension, tachycardia, and a diffuse red macular
rash on her entire body (including palms and soles). She
uses tampons for heavy bleeding. BP = 79/39. Which
organ is responsible for her symptoms?

A) S. aureus - toxic shock syndrome!


B) T. pallidum
Q118 OB/GYN
A pregnant woman at 32-weeks gestation
presents with severe heartburn and RUQ pain.
Her BP = 161/92 (new onset HTN). Platelets =
80,000. Urine dipstick shows 2+ protein. She is
mildly hyperglycemic. What is the cause of her
pain?

A) Fatty infiltration of the liver


B) Distention of the liver capsule
Q118 OB/GYN
A pregnant woman at 32-weeks gestation
presents with severe heartburn and RUQ pain.
Her BP = 161/92 (new onset HTN). Platelets =
80,000. Urine dipstick shows 2+ protein. She is
mildly hyperglycemic. What is the cause of her
pain?

A) Fatty infiltration of the liver


B) Distention of the liver capsule
Q118 OB/GYN
A pregnant woman at 32-weeks gestation
presents with severe heartburn and RUQ pain.
Her BP = 161/92 (new onset HTN). Platelets =
80,000. Urine dipstick shows 2+ protein. She is
mildly hyperglycemic. What is the cause of her
pain?

A) Fatty infiltration of the liver - hypoglycemia


B) Distention of the liver capsule - preeclampsia
with severe features and HELLP
Q119 OB/GYN
A woman with preeclampsia develops dyspnea, hypoxia,
3+ pitting edema of the lower extremities, and
bibasilar crackles. What is the diagnosis?

A) Pulmonary embolus
B) Amniotic fluid embolus
C) Pulmonary edema
Q119 OB/GYN
A woman with preeclampsia develops dyspnea, hypoxia,
3+ pitting edema of the lower extremities, and
bibasilar crackles. What is the diagnosis?

A) Pulmonary embolus
B) Amniotic fluid embolus
C) Pulmonary edema
Q119 OB/GYN
A woman with preeclampsia develops dyspnea, hypoxia,
3+ pitting edema of the lower extremities, and
bibasilar crackles. What is the diagnosis?

C) Pulmonary edema - a life-threatening complication


of preeclampsia with SF; in preeclampsia, the
generalized arterial vasospasm leads to ↑SVR and
↑afterload→↑pulmonary capillary pressure →drives
fluid from the capillaries into the interstitium
-increased capillary permeability also promotes
decreased serum albumin levels (peripheral edema)
Q120 OB/GYN
A 23-year-old woman has hirsutism and acne that started
developing a few months ago. She has also lost 12 lbs. Pelvic
exam shows an enlarged clitoris but no other abnormal
external genitalia. Total testosterone is elevated but DHEAS
is normal. What is the diagnosis?

A) Aromatase deficiency
B) PCOS
C) Sertoli-Leydig cell tumor
Q120 OB/GYN
A 23-year-old woman has hirsutism and acne that started
developing a few months ago. She has also lost 12 lbs. Pelvic
exam shows an enlarged clitoris but no other abnormal
external genitalia. Total testosterone is elevated but DHEAS
is normal. What is the diagnosis?

A) Aromatase deficiency
B) PCOS
C) Sertoli-Leydig cell tumor
Q120 OB/GYN
A 23-year-old woman has hirsutism and acne that started
developing a few months ago. She has also lost 12 lbs. Pelvic
exam shows an enlarged clitoris but no other abnormal
external genitalia. Total testosterone is elevated but DHEAS
is normal. What is the diagnosis?

A) Aromatase deficiency - virilization but ↑DHEAS


B) PCOS - testosterone only mildy ↑, x virilization
C) Sertoli-Leydig cell tumor - virilization, ↑testosterone
Q120 OB/GYN
Q121 OB/GYN
A 26-year-old woman comes in at 25 weeks gestation
and ultrasound shows AFI = 40 cm (normal 6-25). A
tracheoesophageal fistula is visualized. What is this
patient at increased risk for?

A) Preterm labor
B) Placenta accreta
Q121 OB/GYN
A 26-year-old woman comes in at 25 weeks gestation
and ultrasound shows AFI = 40 cm (normal 6-25). A
tracheoesophageal fistula is visualized. What is this
patient at increased risk for?

A) Preterm labor
B) Placenta accreta
Q121 OB/GYN
A 26-year-old woman comes in at 25 weeks gestation
and ultrasound shows AFI = 40 cm (normal 6-25). A
tracheoesophageal fistula is visualized. What is this
patient at increased risk for?

A) Preterm labor - polyhydramnios = ↑risk for


preterm labor/ROM due to uterine overdistention
B) Placenta accreta
Q122 OB/GYN
A 35-year-old woman comes in at 33 weeks
gestation due to RUQ pain that occurs several
times a day. BP = 135/83, BMI = 23. What is the
most likely cause?

A) Stretching of the liver capsule


B) Obstruction of the cystic duct
Q122 OB/GYN
A 35-year-old woman comes in at 33 weeks
gestation due to RUQ pain that occurs several
times a day. BP = 135/83, BMI = 23. What is the
most likely cause?

A) Stretching of the liver capsule


B) Obstruction of the cystic duct
Q122 OB/GYN
A 35-year-old woman comes in at 33 weeks
gestation due to RUQ pain that occurs several
times a day. BP = 135/83, BMI = 23. What is the
most likely cause?

A) Stretching of the liver capsule - ↑BP


B) Obstruction of the cystic duct - symptomatic
cholelithiasis (biliary colic) is common in
pregnancy due to increased gallstones formation
Q123 OB/GYN
A 23-year-old non-pregnant woman comes in for mild sharp
pain in her left lower abdomen which began 3 days ago. She
has no associated symptoms. The cervix appears normal but a
small 3.5 cm nontender mass is palpable in the left adnexa;
ultrasound confirms a left ovarian cyst. What is the next step?

A) Aspiration of the cyst


B) Observation and follow up
Q123 OB/GYN
A 23-year-old non-pregnant woman comes in for mild sharp
pain in her left lower abdomen which began 3 days ago. She
has no associated symptoms. The cervix appears normal but a
small 3.5 cm nontender mass is palpable in the left adnexa;
ultrasound confirms a left ovarian cyst. What is the next step?

A) Aspiration of the cyst


B) Observation and follow up
Q123 OB/GYN
A 23-year-old non-pregnant woman comes in for mild sharp
pain in her left lower abdomen which began 3 days ago. She
has no associated symptoms. The cervix appears normal but a
small 3.5 cm nontender mass is palpable in the left adnexa;
ultrasound confirms a left ovarian cyst. What is the next step?

A) Aspiration of the cyst - rarely indicated (not diagnostic)


B) Observation and follow up - usually resolve spontaneously
Q124 OB/GYN
A month after delivering a baby a 27-year-old woman
presents with a malodorous vaginal discharge. On
pelvic exam, there is a small dark red area on the
posterior vaginal wall with an associated tan brown
discharge. What is the diagnosis?

A) Rectovaginal fistula
B) Vesicovaginal fistula
Q124 OB/GYN
A month after delivering a baby a 27-year-old woman
presents with a malodorous vaginal discharge. On
pelvic exam, there is a small dark red area on the
posterior vaginal wall with an associated tan brown
discharge. What is the diagnosis?

A) Rectovaginal fistula
B) Vesicovaginal fistula
Q124 OB/GYN
A month after delivering a baby a 27-year-old woman presents
with a malodorous vaginal discharge. On pelvic exam, there is a
small dark red area on the posterior vaginal wall with an
associated tan brown discharge. What is the diagnosis?

A) Rectovaginal fistula - tan-brown vaginal discharge


representing fecal material passing through; posterior vaginal
lesion; rectal mucosa may infiltrate; (occur most often after 3rd
of 4th degree lacerations with poor wound repair); tx: surgery
B) Vesicovaginal fistula - watery discharge; anterior fistula
Q125 OB/GYN
A 20-year-old woman 2 months postpartum (with a
complicated labor) has a clear slightly malodorous
vaginal discharge. But she has no other symptoms and
vital signs are normal. There is a small area of
granulation tissue on the anterior vaginal wall. What is
the next step?

A) Bladder dye test


B) Wet mount
Q125 OB/GYN
A 20-year-old woman 2 months postpartum (with a
complicated labor) has a clear slightly malodorous
vaginal discharge. But she has no other symptoms and
vital signs are normal. There is a small area of
granulation tissue on the anterior vaginal wall. What is
the next step?

A) Bladder dye test


B) Wet mount
Q125 OB/GYN
A 20-year-old woman 2 months postpartum (with a
complicated labor) has a clear slightly malodorous
vaginal discharge. But she has no other symptoms and
vital signs are normal. There is a small area of
granulation tissue on the anterior vaginal wall. What is
the next step?
A) Bladder dye test
diagnosis for vesicovaginal fistula; exessive fetal head compression during obstructed
labor causes injury and necrosis to maternal vaginal wall; presents with a continuous
clear vaginal discharge with an abnormally elevated pH (≥4.5) due to urine; tx: surgery
Q126 OB/GYN
Women with short interpregnancy
intervals (i.e., <6-18 months
between delivery and the next
pregnancy) have increased risk of:

A) increased birth weight


B) preterm labor ROM
Q126 OB/GYN
Women with short interpregnancy
intervals (i.e., <6-18 months
between delivery and the next
pregnancy) have increased risk of:

A) increased birth weight


B) preterm labor ROM
Q126 OB/GYN
Women with short interpregnancy
intervals (i.e., <6-18 months
between delivery and the next
pregnancy) have increased risk of:

A) increased birth weight ↓


B) preterm labor ROM - possibly due
to persistent stress/inflammation
Q127 OB/GYN
A 28-year-old woman comes in at 41 weeks
with no complaints. Vital signs are normal.
Ultrasound reveals a vertex fetus and the
single deepest pocket of AF is 1.1 cm. What
is the next step?

A) Induction
B) Visit in 1 week
Q127 OB/GYN
A 28-year-old woman comes in at 41 weeks
with no complaints. Vital signs are normal.
Ultrasound reveals a vertex fetus and the
single deepest pocket of AF is 1.1 cm. What
is the next step?

A) Induction
B) Visit in 1 week
Q127 OB/GYN
A 28-year-old woman comes in at 41 weeks with no complaints. Vital signs are normal.
Ultrasound reveals a vertex fetus and the single deepest pocket of AF is 1.1 cm. What is
the next step?

A) Induction
late/post term ↑maternal complications due to several factors
including uteroplacental insufficiency (e.g., late decelerations
and/or oligohydramnios due to blood shunting away from
kidneys toward brain); if late/post term + oligohydramnios,
delivery baby immediately!
B) Visit in 1 week - only if no oligohydramnios and 42≥ weeks
Q128 OB/GYN
Which of the following is an absolute contraindication
to methotrexate therapy (e.g., for an ectopic
pregnancy)?
A) HIV
B) Anemia
C) Active pulmonary disease
D) Hepatic or renal disease
E) Breastfeeding
F) All of the above
Q128 OB/GYN
Which of the following is an absolute contraindication
to methotrexate therapy (e.g., for an ectopic
pregnancy)?
A) HIV
B) Anemia
C) Active pulmonary disease
D) Hepatic or renal disease
E) Breastfeeding
F) All of the above
Q128 OB/GYN
Which of the following is an absolute contraindication
to methotrexate therapy (e.g., for an ectopic
pregnancy)?
A) HIV - immunosuppression
B) Anemia - MTX can worsen anemia
C) Active pulmonary disease - pulmonary toxicity
D) Hepatic or renal disease - ↓drug clearance/metab.
E) Breastfeeding - transfers to breast milk
F) All of the above - so go with surgical removal!
Q128 OB/GYN
Also,

relative contraindications include:


-an adnexal mass ≥3.5 cm
-the presence of fetal cardiac activity
-a β-hCG level ≥5,000 IU/L

all of these too are associated with an increased risk


of methotrexate treatment failure.
Q129 OB/GYN
A 58-year-old woman has no complaints except
for a recent menstrual period a few days ago
since her menopause 5 years earlier. What is the
next step?

A) Reassurance
B) Biopsy
Q129 OB/GYN
A 58-year-old woman has no complaints except
for a recent menstrual period a few days ago
since her menopause 5 years earlier. What is the
next step?

A) Reassurance
B) Biopsy
Q129 OB/GYN
A 58-year-old woman has no complaints except
for a recent menstrual period a few days ago
since her menopause 5 years earlier. What is the
next step?

A) Reassurance
B) Biopsy - women with postmenopausal bleeding
require pap smear (to evaluate cervical cancer)
and either endometrial biopsy or transvaginal
ultrasound (to evaluate endometrial cancer)
Q129 OB/GYN
Note: In women
A 58-year-old woman has no complaints except
who initially
for a recent menstrual period a few days ago undergo a TVUS,
since her menopause 5 years earlier. What is the those with an
endometrium ≤4
next step? mm require no
additional
evaluation. In
A) Reassurance
contrast, women
B) Biopsy - women with postmenopausal bleeding with an
require pap smear (to evaluate cervical cancer) endometrium >4
mm require an
and either endometrial biopsy or transvaginal endometrial
ultrasound (to evaluate endometrial cancer) biopsy.
Q130 OB/GYN
A woman at 36 weeks gestation undergoes a
nonstress test which is nonreactive. BPP = 8.
What is the next step?

A) Freak out
B) Reassurance
Q130 OB/GYN
A woman at 36 weeks gestation undergoes a
nonstress test which is nonreactive. BPP = 8.
What is the next step?

A) Freak out
B) Reassurance
Q130 OB/GYN
A woman at 36 weeks gestation undergoes a
nonstress test which is nonreactive. BPP = 8.
What is the next step?

A) Freak out
B) Reassurance
Q130 OB/GYN
A woman at 36 weeks gestation undergoes a
nonstress test which is nonreactive. BPP = 8.
What is the next step?
A) Freak out
B) Reassurance
Nonstress test evaluates fetal wellbeing via continuous fetal HR monitoring for 20-40
minutes; the results are considered normal/reactive if ≥2 fetal heart rate accelerations
are present (with normal variability and no decelerations); if nonreactive, can be benign
(e.g., fetal sleep cycle) or potential hypoxia; b/c the test is nonspecific, it requires further
evaluation with BPP; 8≥ is good enough and continue with routine care
Q131 OB/GYN
A pregnant woman had 2 prior consecutive,
painless 2nd trimester lossees. Her cervical
length now (at 14 weeks) is normal (~3 cm). What
is the next step?

A) Reassurance
B) Cerclage
Q131 OB/GYN
A pregnant woman had 2 prior consecutive,
painless 2nd trimester lossees. Her cervical
length now (at 14 weeks) is normal (~3 cm). What
is the next step?

A) Reassurance
B) Cerclage
Q131 OB/GYN
Cervical insufficiency is diagnosed by any 1 of the following
criteria:

• Painless cervical dilation in the current pregnancy OR


• A 2nd-trimester cervical length of ≤2.5 cm plus a prior
preterm delivery OR
• ≥2 prior consecutive, painless, second-trimester losses

Management: cerclage in the first trimester; remove suture at


term to allow vaginal delivery
Q132 OB/GYN
Several days after giving a birth, a 26-year-old
presents with heavy dark red vaginal bleeding
with small clots. PE and labs are normal. What is
the next step?

A) Observation & reassurance


B) Von willebrand testing
C) D&C
Q132 OB/GYN
Several days after giving a birth, a 26-year-old
presents with heavy dark red vaginal bleeding
with small clots. PE and labs are normal. Her
uterus is firm. What is the next step?

A) Observation & reassurance


B) Von willebrand testing
C) D&C
Q132 OB/GYN
Several days after giving a birth, a 26-year-old
presents with heavy dark red vaginal bleeding
with small clots. PE and labs are normal. Her
uterus is firm. What is the next step?

A) Observation & reassurance - lochia


B) Von willebrand testing - anemia, large clots
C) D&C - for retained products; soft, boggy uterus
Q132 OB/GYN
Q133 OB/GYN
A 27-year-old woman at 37 weeks comes to the ED due to severe
abdominal pain; she has had a previous C-section and is scheduled for
another one in 2 weeks. PE shows vaginal bleeding and a palpable,
irregular protuberance in the lower abdomen. FHR tracing is
abnormal. What is the diagnosis?

A) Abruptio placentae
B) Uterine rupture
Q133 OB/GYN
A 27-year-old woman at 37 weeks comes to the ED due to severe
abdominal pain; she has had a previous C-section and is scheduled for
another one in 2 weeks. PE shows vaginal bleeding and a palpable,
irregular protuberance in the lower abdomen. FHR tracing is
abnormal. What is the diagnosis?

A) Abruptio placentae
B) Uterine rupture
Q133 OB/GYN
A 27-year-old woman at 37 weeks comes to the ED due to severe
abdominal pain; she has had a previous C-section and is scheduled for
another one in 2 weeks. PE shows vaginal bleeding and a palpable,
irregular protuberance in the lower abdomen. FHR tracing is
abnormal. What is the diagnosis?

A) Abruptio placentae - abdominal pain/bleeding but no protuberance


B) Uterine rupture - C section is a major risk factor; leaves a weakened
scar that's prone to rupture; an abnormal fetal heart rate tracing is
often the first sign (e.g., bradcardia, variable or late decelerations)
Q134 OB/GYN
A 33-year-old woman comes in 6 weeks after delivery to discuss
contraception. She does not want to gain weight and admits that
she is "not responsible with pills each day". She is generally a
heavy menstrual bleeder. What is the best contraception for her?

A) OCP
B) Copper device
C) Medroxyprogesterone
D) Progestin releasing intrauterine device
Q134 OB/GYN
A 33-year-old woman comes in 6 weeks after delivery to discuss
contraception. She does not want to gain weight and admits that
she is "not responsible with pills each day". She is generally a
heavy menstrual bleeder. What is the best contraception for her?

A) OCP
B) Copper device
C) Medroxyprogesterone
D) Progestin releasing intrauterine device
Q134 OB/GYN
A 33-year-old woman comes in 6 weeks after delivery to discuss
contraception. She does not want to gain weight and admits that
she is "not responsible with pills each day". She is generally a
heavy menstrual bleeder. What is the best contraception for her?

A) OCP - can start at ≥6 weeks but must take consistently


B) Copper device - ↑menstrual bleeding
C) Medroxyprogesterone - weight gain
D) Progestin releasing intrauterine device - highly effective; can
low incidence of systemic effects (e.g., weight gain, mood);
decreases heavy menstrual bleeding
Q135 OB/GYN

Premature ROM + fever + nausea +


vomiting + uterine fundal tenderness =

A) PID
B) Chorioamnionitis
Q135 OB/GYN

Premature ROM + fever + nausea +


vomiting + uterine fundal tenderness =

A) PID
B) Chorioamnionitis
Q135 OB/GYN

Premature ROM + fever + nausea +


vomiting + uterine fundal tenderness =

A) PID - thick mucus; x 3rd trimester


B) Chorioamnionitis - i.e., intraamniotic
infection (risk factor = PROM)
Q136 OB/GYN
What is the next step, besides broad spectrum
antibiotics?

A) C-section
B) Vaginal delivery
C) Tocolytics
Q136 OB/GYN
What is the next step, besides broad spectrum
antibiotics?

A) C-section
B) Vaginal delivery
C) Tocolytics
Q136 OB/GYN
What is the next step, besides broad spectrum
antibiotics?

A) C-section
B) Vaginal delivery
C) Tocolytics
Q136 OB/GYN
What is the next step, besides broad spectrum
antibiotics?

A) C-section
B) Vaginal delivery
C) Tocolytics
Q136 OB/GYN
What is the next step, besides broad spectrum
antibiotics?

A) C-section - no reason for it


B) Vaginal delivery
C) Tocolytics - (e.g., nifedipine, indomethecin)
contraindicated in IAI
Q137 OB/GYN
A 30-year-old woman comes in 6 weeks after labor
for a postpartum visit. Her pregnancy was
complicated by gestational DM and preeclampsia
with severe features. What is the next best step?

A) Endometrial biopsy
B) 2 hour oral glucose tolerance test
C) 24-hour urine protein collection
Q137 OB/GYN
A 30-year-old woman comes in 6 weeks after labor
for a postpartum visit. Her pregnancy was
complicated by gestational DM and preeclampsia
with severe features. What is the next best step?

A) Endometrial biopsy
B) 2 hour oral glucose tolerance test
C) 24-hour urine protein collection
Q137 OB/GYN
A 30-year-old woman comes in 6 weeks after labor
for a postpartum visit. Her pregnancy was
complicated by gestational DM and preeclampsia
with severe features. What is the next best step?

A) Endometrial biopsy
B) 2 hour oral glucose tolerance test
-screening 6-12 weeks postpartum
C) 24-hour urine protein collection
-urine screening during, not after pregnancy
Q138 OB/GYN
Which Rh-D negative woman requires anti-D
immunoglobulin?

A) Patient at 28-32 weeks


B) Within 72 hours of delivery of RhD+ baby
C) Within 72 hours after spontaneous abortion
D) 2nd & 3rd trimester bleeding
E) All of the above
Q138 OB/GYN
Which Rh-D negative woman requires anti-D
immunoglobulin?

A) Patient at 28-32 weeks


B) Within 72 hours of delivery of RhD+ baby
C) Within 72 hours after spontaneous abortion
D) 2nd & 3rd trimester bleeding
E) All of the above
Q139 OB/GYN
PID with high fever and vomiting is managed with:

A) Inpatient treatment with doxycycline & cephalosporin


B) Outpatient treatment with doxycycline & cephalosporin
Q139 OB/GYN
PID with high fever and vomiting is managed with:

A) Inpatient treatment with doxycycline & cephalosporin


B) Outpatient treatment with doxycycline & cephalosporin
Q139 OB/GYN
PID with high fever and vomiting is managed with:

A) Inpatient treatment with doxycycline & cephalosporin


-PID indications for inpatient treatment:
• Pregnancy
• Inability to tolerate oral meds
• Noncompliant with therapy
• Severe presentation (e.g., fever, vomiting)
• Complications (eg, tuboovarian abscess)
Q140 OB/GYN
Smelly, thin, yellow green vaginal discharge,
vaginal erythema, elevated vaginal pH (>4.5) =

A) Clue cells on microscopy


B) Flagellated organisms on microscopy
Q140 OB/GYN
Smelly, thin, yellow green vaginal discharge,
vaginal erythema, elevated vaginal pH (>4.5) =

A) Clue cells on microscopy


B) Flagellated organisms on microscopy
Q140 OB/GYN
Smelly, thin, yellow green vaginal discharge,
vaginal erythema, elevated vaginal pH (>4.5) =

A) Clue cells on microscopy - Gardnerella; gray-


white discharge; no erythema
B) Flagellated organisms on microscopy
Q140 OB/GYN
Q141 OB/GYN
Neonatal lupus involves primarily cutaneous
(e.g., periorbital or scalp rash) and:

A) renal findings
B) cardiac findings
Q141 OB/GYN
Neonatal lupus involves primarily cutaneous
(e.g., periorbital or scalp rash) and:

A) renal findings
B) cardiac findings
Q141 OB/GYN
Neonatal lupus involves primarily cutaneous
(e.g., periorbital or scalp rash) and:

A) renal findings
B) cardiac findings -
e.g., fetal AV block (develops at 18-24 weeks
gestation), as maternal antibodies bind to fetal
cardiac cells, damaging the AV node (shows up
on FHR tracing as persistent bradycardia)
Q142 OB/GYN
A woman has irregular menses + ↑testosterone +
LH/FSH imbalance. Why is she infertile?

A) Failure of the follicle to mature


B) Primary ovarian insufficiency
Q142 OB/GYN
A woman has irregular menses + ↑testosterone +
LH/FSH imbalance. Why is she infertile?

A) Failure of the follicle to mature


B) Primary ovarian insufficiency
Q142 OB/GYN
A woman has irregular menses + ↑testosterone +
LH/FSH imbalance. Why is she infertile?

A) Failure of the follicle to mature - PCOS;


persistently ↑estrone levels → high freq. short
GnRH impulses → ↑LH/FSH ratio → x LH surge →
failure of follicle to mature
B) Primary ovarian insufficiency - normal
testosterone
Q143 OB/GYN
Patient says "I am sorry doctor, but I have
made up my mind, I am having a home
birth". What should you say?

A) I don't recommend home births


because you are putting your baby at risk
B) Let's discuss scenarios in which you
should come to the hospital when
delivering at home
Q143 OB/GYN
Patient says "I am sorry doctor, but I have
made up my mind, I am having a home
birth". What should you say?

A) I don't recommend home births


because you are putting your baby at risk
B) Let's discuss scenarios in which you
should come to the hospital when
delivering at home
Q143 OB/GYN
Patient says "I am sorry doctor, but I have
made up my mind, I am having a home
birth". What should you say?

A) I don't recommend home births


because you are putting your baby at risk
B) Let's discuss scenarios in which you
should come to the hospital when
delivering at home - must balance safety
with patient autonomy
Q144 OB/GYN
Patient says "I am sorry doctor, but I have
made up my mind, I am having a home
birth". What should you say?

A) I don't recommend home births


because you are putting your baby at risk
B) Let's discuss scenarios in which you
should come to the hospital when
delivering at home
Q145 OB/GYN
What is going on with
Curve D (green)?

A) Normal labor
B) Active phase arrest
Q145 OB/GYN
What is going on with
Curve D (green)?

A) Normal labor
B) Active phase arrest
Q145 OB/GYN
What is going on with
Curve D (green)?

A) Normal labor
B) Active phase arrest
active phase arrest is no cervical
change in ≥4 hours with adequate
contractions, or no change in ≥6
hours with inadequate
contractions. Labor arrest is
managed by C section.
Q146 OB/GYN
What is the first-line treatment for migraines
during pregnancy?

A) NSAIDs
B) Ergotamine
C) Acetaminophen
Q146 OB/GYN
What is the first-line treatment for migraines
during pregnancy?

A) NSAIDs
B) Ergotamine
C) Acetaminophen
Q146 OB/GYN
What is the first-line treatment for migraines
during pregnancy?

A) NSAIDs - 2nd line; not in 1st/3rd trimesters


B) Ergotamine - x during pregnancy due to risk
of ↑uterine constriction
C) Acetaminophen - opioids, antiemetics, and
NSAIDs (2nd trimester) may be 2nd line
Q147 OB/GYN
Third trimester miscarriage + ↑total bile acids
+ generalized pruritis + platelets + 140,000 =

A) Intrahepatic cholestasis
B) HELLP
Q147 OB/GYN
Third trimester miscarriage + ↑total bile acids
+ generalized pruritis + platelets + 140,000 =

A) Intrahepatic cholestasis
B) HELLP
Q147 OB/GYN
Third trimester miscarriage + ↑total bile acids
+ generalized pruritis + platelets + 140,000 =

A) Intrahepatic cholestasis - pruritis worse on


hands and feet; occurs in 3rd trimester as
↑estrogen/progestorone → biliary stasis; bile
acid crosses placenta → miscarriage
B) HELLP - platelets below 100,000
Q147 OB/GYN
Third trimester miscarriage + ↑total bile acids
+ generalized pruritis + platelets + 140,000 =

A) Intrahepatic cholestasis - pruritis worse on Management:


hands and feet; occurs in 3rd trimester as -Ursodeoxycholic acid
↑estrogen/progestorone → biliary stasis; bile -Antihistamines
acid crosses placenta → miscarriage -Delivery at 37 weeks
B) HELLP - platelets below 100,000 gestation
Q148 OB/GYN
A 38-year-old woman complains of urinary
incontinence. She was diagnosed with MS
several years ago. What is the mechanism?

A) Weakness of pelvic floor muscles


B) Demyelination
C) Muscle overactivity
Q148 OB/GYN
A 38-year-old woman complains of urinary
incontinence. She was diagnosed with MS
several years ago. What is the mechanism?

A) Weakness of pelvic floor muscles


B) Demyelination
C) Muscle overactivity
Q148 OB/GYN
A 38-year-old woman complains of urinary
incontinence. She was diagnosed with MS
several years ago. What is the mechanism?

A) Weakness of pelvic floor muscles


B) Demyelination - would result in underactivity
C) Muscle overactivity - i.e., detrusor; due to loss
of upper motor neuron inhibition of detrusor
contraction; "urge incontinence"
Q149 OB/GYN
Which woman CAN have a vaginal delivery?

A) A woman with history of extensive myomectomy


B) A woman with a history of a vertical C section
C) A woman with a history of a horizontal C section
Q149 OB/GYN
Which woman CAN have a vaginal delivery?

A) A woman with history of extensive myomectomy


B) A woman with a history of a vertical C section
C) A woman with a history of a horizontal C section
Q150 OB/GYN
A 20-year-old woman who hasn't seen a doctor in 15
years comes to meet with an OBGYN. What is
recommended?

A) HPV vaccine
B) Pap smear
C) Both
Q150 OB/GYN
A 20-year-old woman who hasn't seen a doctor in 15
years comes to meet with an OBGYN. What is
recommended?

A) HPV vaccine
B) Pap smear
C) Both
Q150 OB/GYN
A 20-year-old woman who hasn't seen a doctor in 15
years comes to meet with an OBGYN. What is
recommended?

A) HPV vaccine - ages 11-26 (but can be given 9-45)


B) Pap smear - testing begins at 21 in
immunocompetent patients (+/- sex activity)
C) Both
Q150 OB/GYN
A 20-year-old woman who hasn't seen a doctor in 15
years comes to meet with an OBGYN. What is
recommended?

A) HPV vaccine - ages 11-26 (but can be given 9-45)


Note: HPV vaccine also not indicated in pregnancy
B) Pap smear - testing begins at 21 in
immunocompetent patients (+/- sex activity)
Q151 OB/GYN
It is discovered through testing that a
fetus has bilateral renal agenesis. The
fetus is in breech position and mom is
going into labor. How should the baby be
delivered?

A) C-section
B) Vaginally
Q151 OB/GYN
It is discovered through testing that a
fetus has bilateral renal agenesis. The
fetus is in breech position and mom is
going into labor. How should the baby be
delivered?

A) C-section
B) Vaginally
Q151 OB/GYN
It is discovered through testing that a fetus
has bilateral renal agenesis. The fetus is in
breech position and mom is going into labor.
How should the baby be delivered?

A) C-section
B) Vaginally - with lethal fetal anomalies (e.g.,
BRA, anencephaly, holoprosencephaly)
minimize maternal morbidity and mortality
Q152 OB/GYN
A 15-year-old girl hasn't had a period for 4 months.
Her blood pressure is 152/93. There is a nontender
palpable mass in the lower abdomen. Serum hCG is
elevated. What is the diagnosis?

A) Embryonal carcinoma
B) Hydatidiform mole
Q152 OB/GYN
A 15-year-old girl hasn't had a period for 4 months.
Her blood pressure is 152/93. There is a nontender
palpable mass in the lower abdomen. Serum hCG is
elevated. What is the diagnosis?

A) Embryonal carcinoma
B) Hydatidiform mole
Q152 OB/GYN
A 15-year-old girl hasn't had a period for 4 months.
Her blood pressure is 152/93. There is a nontender
palpable mass in the lower abdomen. Serum hCG is
elevated. What is the diagnosis?

A) Embryonal carcinoma
B) Hydatidiform mole - HM can present with
preeclampsia with SF at <20 weeks gestation
Q153 OB/GYN
What should be done for a woman with heavy
ovulatory menstrual bleeding but no plans for
future fertility (and low risk for endometrial
malignancy)?

A) Hysterosalpingography
B) Endometrial ablation
Q153 OB/GYN
What should be done for a woman with heavy
ovulatory menstrual bleeding but no plans for
future fertility (and low risk for endometrial
malignancy)?

A) Hysterosalpingography
B) Endometrial ablation
Q153 OB/GYN
What should be done for a woman with heavy
ovulatory menstrual bleeding but no plans for
future fertility (and low risk for endometrial
malignancy)?

A) Hysterosalpingography - detects cause of HMB


B) Endometrial ablation - minimally invasive;
destroys/resects endometrial lining
Q153 OB/GYN
Q154 OB/GYN
During induced labor at 36-weeks (for preeclampsia
with SF), a woman is given prophylactic antibiotics.
She develops dyspnea and wheezes, hypotension,
and tachycardia, and she appears flushed. What is
the cause?

A) Anaphylaxis
B) Amniotic fluid embolism
Q154 OB/GYN
During induced labor at 36-weeks (for preeclampsia
with SF), a woman is given prophylactic antibiotics.
She develops dyspnea and wheezes, hypotension,
and tachycardia, and she appears flushed. What is
the cause?

A) Anaphylaxis
B) Amniotic fluid embolism
Q154 OB/GYN
During induced labor at 36-weeks (for preeclampsia
with SF), a woman is given prophylactic antibiotics.
She develops dyspnea and wheezes, hypotension,
and tachycardia, and she appears flushed. What is
the cause?

A) Anaphylaxis - e.g., stings, meds, nuts


B) Amniotic fluid embolism - ass. w/ DIC; crackles
Q155 OB/GYN
Fetal ultrasound of a woman at 26 weeks reveals bilateral
ventriculomegaly and multiple intracranial calcifications
in the basal ganglia. Hepatosplenomegaly is also present.
What is the diagnosis?

A) T gondii
B) Parvovirus B19
C) HSV
D) Listeria
Q155 OB/GYN
Fetal ultrasound of a woman at 26 weeks reveals bilateral
ventriculomegaly and multiple intracranial calcifications
in the basal ganglia. Hepatosplenomegaly is also present.
What is the diagnosis?

A) T gondii
B) Parvovirus B19
C) HSV
D) Listeria
Q155 OB/GYN

Fetal ultrasound of a woman at 26 weeks reveals bilateral


ventriculomegaly and multiple intracranial calcifications
in the basal ganglia. Hepatosplenomegaly is also present.
What is the diagnosis?

A) T gondii - hydrocephalus, intracranial calcifications


B) Parvovirus B19 - fetal hydrops (x intracranial calcific.)
C) HSV - mothers have painful ulcers; no brain calcific.
D) Listeria - disseminated abscesses
Q156 OB/GYN
A woman has a bicornuate uterus. Which form
of contraception is indicated?

A) Copper IUD
B) Depot medroxyprogesterone
Q156 OB/GYN
A woman has a bicornuate uterus. Which form
of contraception is indicated?

A) Copper IUD
B) Depot medroxyprogesterone
Q156 OB/GYN
A woman has a bicornuate uterus. Which form
of contraception is indicated?

A) Copper IUD - doesn't work in women with


distorted uterine anatomy (e.g., bicornuate
uterus, large fibroids)
B) Depot medroxyprogesterone
Q157 OB/GYN
What is first-line prevention for migraines in
pregnancy?

A) Sumatriptan
B) Topiramate
C) Propranolol
Q157 OB/GYN
What is first-line prevention for migraines in
pregnancy?

A) Sumatriptan
B) Topiramate
C) Propranolol
Q157 OB/GYN
What is first-line prevention for migraines in
pregnancy?

A) Sumatriptan - abort migraines, not prevention


B) Topiramate - fetal abnormalities
C) Propranolol - safe for babby (also CCB's)
Q158 OB/GYN
What is the greatest risk factor for shoulder
dystocia?

A) Hypertension
B) Short interpregnancy interval
C) Maternal weight
Q158 OB/GYN
What is the greatest risk factor for shoulder
dystocia?

A) Hypertension
B) Short interpregnancy interval
C) Maternal weight
Q158 OB/GYN
What is the greatest risk factor for shoulder
dystocia?

A) Hypertension - fetal growth restriction


B) Short interpregnancy interval - rupture
C) Maternal weight
Q159 OB/GYN
A 27-year-old female smoker at 23-weeks gestation
comes to the ED with vaginal bleeding that began
after intercourse. The bleeding continues to soak
but doesn't hurt. Fetal monitoring is reassuring.
What is the diagnosis?

A) Cervical insufficiency
B) Placenta previa
Q159 OB/GYN
A 27-year-old female smoker at 23-weeks gestation
comes to the ED with vaginal bleeding that began
after intercourse. The bleeding continues to soak
but doesn't hurt. Fetal monitoring is reassuring.
What is the diagnosis?

A) Cervical insufficiency
B) Placenta previa
Q159 OB/GYN
A 27-year-old female smoker at 23-weeks gestation
comes to the ED with vaginal bleeding that began
after intercourse. The bleeding continues to soak
but doesn't hurt. Fetal monitoring is reassuring.
What is the diagnosis?

A) Cervical insufficiency - miscarriage; x blood


B) Placenta previa - painless vaginal bleeding
Q160 OB/GYN
An 8-year-old girl has had a
malodorous vaginal discharge for
several days secondary to retained
toilet paper.

A) Speculum examination
B) Anesthetic gel and irrigation
with warm water
Q160 OB/GYN
An 8-year-old girl has had a
malodorous vaginal discharge for
several days secondary to retained
toilet paper.

A) Speculum examination
B) Anesthetic gel and irrigation
with warm water
Q160 OB/GYN
An 8-year-old girl has had a malodorous vaginal
discharge for several days secondary to retained
toilet paper.

A) Speculum examination - too narrow; vaginoscopy


B) Anesthetic gel and irrigation with warm water -
irrigation for removal
Q160 OB/GYN
An 8-year-old girl has had a malodorous vaginal
discharge for several days secondary to retained
toilet paper.

A) Speculum examination - too narrow; vaginoscopy


B) Anesthetic gel and irrigation with warm water -
irrigation for removal
Q161 OB/GYN
During a tubal ligation, it is discovered
that an (asymptomatic) woman has
multiple intraabdominal lesions;
pathology reveals endometrial glands,
stroma, and hemosiderin laden
macrophages. What is the next step?

A) Combined oral contraceptives


B) Reassurance and observation
Q161 OB/GYN
During a tubal ligation, it is discovered
that an (asymptomatic) woman has
multiple intraabdominal lesions;
pathology reveals endometrial glands,
stroma, and hemosiderin laden
macrophages. What is the next step?

A) Combined oral contraceptives


B) Reassurance and observation
Q161 OB/GYN
During a tubal ligation, it is discovered
that an (asymptomatic) woman has
multiple intraabdominal lesions;
pathology reveals endometrial glands,
stroma, and hemosiderin laden
macrophages. What is the next step?

A) Combined oral contraceptives


B) Reassurance and observation - no
need to treat if asymptomatic
Q162 OB/GYN
Patient with endometrial hyperplasia (who
desire future fertility) are treated with:

A) Clomiphene
B) Endometrial ablation
C) Progestin-releasing IUD
Q162 OB/GYN
Patient with endometrial hyperplasia (who
desire future fertility) are treated with:

A) Clomiphene
B) Endometrial ablation
C) Progestin-releasing IUD
Q163 OB/GYN
A 53-year-old woman complains of vulvar pruritis (with
erythema) for a week. This has happened several times
over the last few months. Microscopy reveals
pseudohyphae. Which test should be performed?

A) Vulvar biopsy
B) HA1c
C) FSH level
Q163 OB/GYN
A 53-year-old woman complains of vulvar pruritis (with
erythema) for a week. This has happened several times
over the last few months. Microscopy reveals
pseudohyphae. Which test should be performed?

A) Vulvar biopsy
B) HA1c
C) FSH level
Q164 OB/GYN
A 2 week post-partum patient has new-onset
seizures, papilledema, and headache. She has no
hypertension or proteinuria and CT of the head is
normal. What is the next best step?

A) Magnesium
B) MR venography of the brain
Q164 OB/GYN
A 2 week post-partum patient has new-onset
seizures, papilledema, and headache. She has no
hypertension or proteinuria and CT of the head is
normal. What is the next best step?

A) Magnesium
B) MR venography of the brain
Q164 OB/GYN
A 2 week post-partum patient has new-onset
seizures, papilledema, and headache. She has no
hypertension or proteinuria and CT of the head is
normal. What is the next best step?

A) Magnesium - preeclampsia can present


after birth, but BP would be ↑
B) MR venography of the brain - ICH
Q165 OB/GYN
A woman with menstrual irregularities and enlarged
ovaries on exam complains of infertility. Which
medication can help her with ovulation induction?

A) Cyclic progestorone
C) Letrozole
Q165 OB/GYN
A woman with menstrual irregularities and enlarged
ovaries on exam complains of infertility. Which
medication can help her with ovulation induction?

A) Cyclic progestorone
C) Letrozole
Q165 OB/GYN
A woman with menstrual irregularities and enlarged
ovaries on exam complains of infertility. Which
medication can help her with ovulation induction?

A) Cyclic progesterone - endometrial protection


C) Letrozole - aromatase inhibitor; induces ovulation
in patients with PCOS
Q166 OB/GYN
An 87-year-old woman from a nursing home is
evaluated for PMB. Besides an edematous, tender
vulva, PE is normal and on ultrasound the uterus is
small with a 3-mm endometrial lining; no adnexal
masses are present. What is the next step?

A) Biopsy
B) Sex abuse screening
Q166 OB/GYN
An 87-year-old woman from a nursing home is
evaluated for PMB. Besides an edematous, tender
vulva, PE is normal and on ultrasound the uterus is
small with a 3-mm endometrial lining; no adnexal
masses are present. What is the next step?

A) Biopsy
B) Sex abuse screening
Q166 OB/GYN
An 87-year-old woman from a nursing home is
evaluated for PMB. Besides an edematous, tender
vulva, PE is normal and on ultrasound the uterus is
small with a 3-mm endometrial lining; no adnexal
masses are present. What is the next step?

A) Biopsy
B) Sex abuse screening - report to APS
Q167 OB/GYN
Pregnant women have the right to refuse treatment
(e.g., emergency C-section) as nonpregnant patients,
even if the decision does not represent the best
interests of the fetus. T/F.

A) True
B) False
Q167 OB/GYN
Pregnant women have the right to refuse treatment
(e.g., emergency C-section) as nonpregnant patients,
even if the decision does not represent the best
interests of the fetus. T/F.

A) True
B) False
Q167 OB/GYN
Pregnant women have the right to refuse treatment
(e.g., emergency C-section) as nonpregnant patients,
even if the decision does not represent the best
interests of the fetus. T/F.

A) True - until delivery, the mother has complete


autonomy for medical decisions affecting both
herself and the fetus
B) False
Q168 OB/GYN

A 64-year-old woman has PMB and a 1


cm-ulcerated lesion in the posterior
vaginal wall. Endometrial stripe is
3mm. What is the next step?

A) Endometrial biopsy
B) Vaginal biopsy
Q168 OB/GYN

A 64-year-old woman has PMB and a 1


cm-ulcerated lesion in the posterior
vaginal wall. Endometrial stripe is
3mm. What is the next step?

A) Endometrial biopsy
B) Vaginal biopsy
Q168 OB/GYN

A 64-year-old woman has PMB and a 1


cm-ulcerated lesion in the posterior
vaginal wall. Endometrial stripe is
3mm. What is the next step?

A) Endometrial biopsy
B) Vaginal biopsy - concerning for
vaginal SCC
Q169 OB/GYN

A woman with previously regular menses has


not had a period in 4 months. What is the next
step?

A) Pregnancy test
B) Serum FSH, TSH, prolactin levels
Q169 OB/GYN

A woman with previously regular menses has


not had a period in 4 months. What is the next
step?

A) Pregnancy test
B) Serum FSH, TSH, prolactin levels
Q169 OB/GYN

A woman with previously regular menses has


not had a period in 4 months. What is the next
step?

A) Pregnancy test- check this first


B) Serum FSH, TSH, prolactin levels - check
these next!
Q170 OB/GYN
A woman undergoes cervical
conization for CIN 3. Pathology
report shows CIN 3 with all surgical
margins free of neoplasia. What is
the next best step?

A) Pap testing alone in 3 years


B) Pap and HPV cotesting at 1 and 2
years
Q170 OB/GYN
A woman undergoes cervical
conization for CIN 3. Pathology
report shows CIN 3 with all surgical
margins free of neoplasia. What is
the next best step?

A) Pap testing alone in 3 years


B) Pap and HPV cotesting at 1 and 2
years
Q170 OB/GYN
A woman undergoes cervical
conization for CIN 3. Pathology
report shows CIN 3 with all surgical
margins free of neoplasia. What is
the next best step?

A) Pap testing alone in 3 years


B) Pap and HPV cotesting at 1 and 2
years - she's at ↑risk; but if +
margins, need further surgery
Q171 OB/GYN
A woman with severe hyperemesis gravidarum
develops altered mental status, oculomotor
dysfunction, and gait ataxia. What is the
treatment?

A) Penicillin
B) Thiamine followed by glucose
Q171 OB/GYN
A woman with severe hyperemesis gravidarum
develops altered mental status, oculomotor
dysfunction, and gait ataxia. What is the
treatment?

A) Penicillin
B) Thiamine followed by glucose
Q171 OB/GYN
A woman with severe hyperemesis gravidarum
develops altered mental status, oculomotor
dysfunction, and gait ataxia. What is the
treatment?

A) Penicillin - for tabes dorsalis


B) Thiamine followed by glucose - for W.E.
Q172 OB/GYN
10 days after delivering her baby (complicated by 2nd
degree perineal laceration), a woman develops 2 days of
high fever, hypotension, and a diffuse macular rash. On
pelvic exam the perineal laceration is tender. What is the
best treatment?

A) Ceftriaxone
B) Vancomycin + clindamycin
Q172 OB/GYN
10 days after delivering her baby (complicated by 2nd
degree perineal laceration), a woman develops 2 days of
high fever, hypotension, and a diffuse macular rash. On
pelvic exam the perineal laceration is tender. What is the
best treatment?

A) Ceftriaxone
B) Vancomycin + clindamycin
Q172 OB/GYN
10 days after delivering her baby (complicated by 2nd
degree perineal laceration), a woman develops 2 days of
high fever, hypotension, and a diffuse macular rash. On
pelvic exam the perineal laceration is tender. What is the
best treatment?

A) Ceftriaxone - for gram negative infections


B) Vancomycin + clindamycin - for Toxic shock syndrome
(caused by S. aureus bacteremia)
Q173 OB/GYN
A 19-year-old pregnant woman presents with
nausea, high fever, and unilateral flank pain. Both
mom and fetus are tachycardic. Uterus is
nontender. What is the diagnosis?

A) Acute pyelonephritis
B) Intraamniotic infection
Q173 OB/GYN
A 19-year-old pregnant woman presents with
nausea, high fever, and unilateral flank pain. Both
mom and fetus are tachycardic. Uterus is
nontender. What is the diagnosis?

A) Acute pyelonephritis
B) Intraamniotic infection
Q173 OB/GYN
A 19-year-old pregnant woman presents with
nausea, high fever, and unilateral flank pain. Both
mom and fetus are tachycardic. Uterus is
nontender. What is the diagnosis?

A) Acute pyelonephritis - flank tenderness makes


intraamniotic infection more likely
B) Intraamniotic infection
Q174 OB/GYN
A woman has leakage of urine,
dyspareunia, and a tender anterior
vaginal wall mass (which bleeds upon
palpation). What is going on?

A) Tract between bladder and vagina


B) Outpouching of the urethral meatus
Q174 OB/GYN
A woman has leakage of urine,
dyspareunia, and a tender anterior
vaginal wall mass (which bleeds upon
palpation). What is going on?

A) Tract between bladder and vagina


B) Outpouching of the urethral meatus
Q174 OB/GYN
A woman has leakage of urine,
dyspareunia, and a tender anterior
vaginal wall mass (which bleeds upon
palpation). What is going on?

A) Tract between bladder and vagina


B) Outpouching of the urethral meatus -
urethral diverticulum
Q175 OB/GYN

Which medication is used to prevent


preeclampsia onset in high risk
patients?

A) Nifedipine
B) Aspirin
Q175 OB/GYN

Which medication is used to prevent


preeclampsia onset in high risk
patients?

A) Nifedipine
B) Aspirin
Q175 OB/GYN

Which medication is used to prevent


preeclampsia onset in high risk
patients?

A) Nifedipine
B) Aspirin - low dose aspirin at
12-28 weeks (but better before 16 )
Q176 OB/GYN

An HIV pregnant woman at 37 weeks


gestation is having contractions with
cervical dilation. Her viral load is 5000. How
should the baby be delivered?

A) Vaginally
B) C-section
Q176 OB/GYN

An HIV pregnant woman at 37 weeks


gestation is having contractions with
cervical dilation. Her viral load is 5000. How
should the baby be delivered?

A) Vaginally
B) C-section
Q176 OB/GYN
An HIV pregnant woman at 37 weeks
gestation is having contractions with
cervical dilation. Her viral load is 5000. How
should the baby be delivered?

A) Vaginally - only if VL ≤1000 (x zidovidine)


B) C-section - ↓perinatal HIV transmission &
zidovudine
Q177 OB/GYN

A 39-year-old woman at 36 weeks gestation


with no significant medical history presents
with BP = 170/95 and 3+ protein in the urine.
Which condition is she at ↑ risk for?

A) Subarachnoid hemorrhage
B) Acute ischemic stroke
Q177 OB/GYN

A 39-year-old woman at 36 weeks gestation


with no significant medical history presents
with BP = 170/95 and 3+ protein in the urine.
Which condition is she at ↑ risk for?

A) Subarachnoid hemorrhage
B) Acute ischemic stroke
Q177 OB/GYN

A 39-year-old woman at 36 weeks gestation


with no significant medical history presents
with BP = 170/95 and 3+ protein in the urine.
Which condition is she at ↑ risk for?

A) Subarachnoid hemorrhage
B) Acute ischemic stroke - preeclampsia
Q178 OB/GYN
A 28-year-old woman at 26 weeks has had intermittent
leakage of fluid for the past 7 hours; on spec exam, purulent
amniotic fluid emerges from the cervix. Temp = 103F and
pulse is 116/min. The uterus is tender to palpation. Besides
antibiotics, what should be done?

A) Amnioinfusion
B) Fetal lung maturity testing
C) Immediate induction
Q178 OB/GYN
A 28-year-old woman at 26 weeks has had intermittent
leakage of fluid for the past 7 hours; on spec exam, purulent
amniotic fluid emerges from the cervix. Temp = 103F and
pulse is 116/min. The uterus is tender to palpation. Besides
antibiotics, what should be done?

A) Amnioinfusion
B) Fetal lung maturity testing
C) Immediate induction
Q178 OB/GYN
A 28-year-old woman at 26 weeks has had intermittent
leakage of fluid for the past 7 hours; on spec exam, purulent
amniotic fluid emerges from the cervix. Temp = 103F and
pulse is 116/min. The uterus is tender to palpation. Besides
antibiotics, what should be done?

A) Amnioinfusion
B) Fetal lung maturity testing
C) Immediate induction - PROM + infection
Q179 OB/GYN
A 13-year-old girl is evaluated for acne which began several
months ago; she has had no menstrual period yet and doesn't
have breast bud development. Pelvic exam shows the clitoris
protruding from the clitoral hood and bilateral labial
masses. What is the diagnosis?

A) 5-alpha-reductase deficiency
B) Androgen insensitivity syndrome
C) Sertoli-Leydig cell tumor
Q179 OB/GYN
A 13-year-old girl is evaluated for acne which began several
months ago; she has had no menstrual period yet and doesn't
have breast bud development. Pelvic exam shows the clitoris
protruding from the clitoral hood and bilateral labial
masses. What is the diagnosis?

A) 5-alpha-reductase deficiency
B) Androgen insensitivity syndrome
C) Sertoli-Leydig cell tumor
Q179 OB/GYN
A 13-year-old girl is evaluated for acne which began several
months ago; she has had no menstrual period yet and doesn't
have breast bud development. Pelvic exam shows the clitoris
protruding from the clitoral hood and bilateral labial
masses. What is the diagnosis?

A) 5-alpha-reductase deficiency - no breasts


B) Androgen insensitivity syndrome - breasts
C) Sertoli-Leydig cell tumor - virilization
Q180 OB/GYN
A pregnant woman at 14 weeks gestation has a
BP = 145/96. Which obstetric complication is
she at risk for?

A) PPROM
B) Preterm delivery
Q180 OB/GYN
A pregnant woman at 14 weeks gestation has a
BP = 145/96. Which obstetric complication is
she at risk for?

A) PPROM
B) Preterm delivery
Q180 OB/GYN
A pregnant woman at 14 weeks gestation has a
BP = 145/96. Which obstetric complication is
she at risk for?

A) PPROM
B) Preterm delivery - chronic hypertension
also a risk for FGR & superimposed
preeclampsia
Q181 OB/GYN
A woman in her first trimester develops a
white/gray malodorous vaginal discharge, and
labs reveal clue cells. What is the next step?

A) Oral metronidazole before delivery


B) Oral metronidazole now
C) Topical fluconazole
Q181 OB/GYN
A woman in her first trimester develops a
white/gray malodorous vaginal discharge, and
labs reveal clue cells. What is the next step?

A) Oral metronidazole before delivery


B) Oral metronidazole now
C) Topical fluconazole
Q181 OB/GYN
A woman in her first trimester develops a
white/gray malodorous vaginal discharge, and
labs reveal clue cells. What is the next step?

A) Oral metronidazole before delivery


B) Oral metronidazole now - or clindamycin
C) Topical fluconazole
Q182 OB/GYN
A woman goes into labor at 25 weeks. She is
given indomethacin tocolysis to inhibit
contractions. What is she at risk for?

A) Polyhydramnios
B) Oligohydramnios
Q182 OB/GYN
A woman goes into labor at 25 weeks. She is
given indomethacin tocolysis to inhibit
contractions. What is she at risk for?

A) Polyhydramnios
B) Oligohydramnios
Q182 OB/GYN
A woman goes into labor at 25 weeks. She is
given indomethacin tocolysis to inhibit
contractions. What is she at risk for?

A) Polyhydramnios
B) Oligohydramnios (indomethacin can cause
oligohyadramnios and premature closure of
the DA, although benefits > risks)
Q183 OB/GYN
A 39-year-old woman hasn't had her period in 2 months.
Pelvic exam shows a 12-week-size uterus and bilateral
adnexal masses; ultrasound shows a uterus filled with
multiple small cysts but no embryo; the ovaries have a
multilocular cystic appearance. beta-hCG is elevated.
What does she have?

A) Benign cystic teratomas


B) Theca lutein cysts
Q183 OB/GYN
A 39-year-old woman hasn't had her period in 2 months.
Pelvic exam shows a 12-week-size uterus and bilateral
adnexal masses; ultrasound shows a uterus filled with
multiple small cysts but no embryo; the ovaries have a
multilocular cystic appearance. beta-hCG is elevated.
What does she have?

A) Benign cystic teratomas


B) Theca lutein cysts
Q183 OB/GYN
A 39-year-old woman hasn't had her period in 2 months.
Pelvic exam shows a 12-week-size uterus and bilateral
adnexal masses; ultrasound shows a uterus filled with
multiple small cysts but no embryo; the ovaries have a
multilocular cystic appearance. beta-hCG is elevated.
What does she have?

A) Benign cystic teratomas


B) Theca lutein cysts - may be seen in hydatidiform mole
Q184 OB/GYN
A woman without children has signs or early pregnancy
(e.g., morning sickness and amenorrhea) but evaluation
excludes pregnancy. What is the diagnosis?

A) Missed abortion
B) Pseudocyesis
Q184 OB/GYN
A woman without children has signs or early pregnancy
(e.g., morning sickness and amenorrhea) but evaluation
excludes pregnancy. What is the diagnosis?

A) Missed abortion
B) Pseudocyesis
Q184 OB/GYN
A woman without children has signs or early pregnancy
(e.g., morning sickness and amenorrhea) but evaluation
excludes pregnancy. What is the diagnosis?

A) Missed abortion
B) Pseudocyesis - persisent non-delusional belief of
being pregnant in a nonpregnant patient
Q185 OB/GYN
How are uncomplicated perineal lacerations
(e.g., no fever, purulence) managed (e.g.,
perineal pain, particularly with voiding)?

A) NSAID's and sitz baths


B) Antibiotics
Q185 OB/GYN
How are uncomplicated perineal lacerations
(e.g., no fever, purulence) managed (e.g.,
perineal pain, particularly with voiding)?

A) NSAID's and sitz baths


B) Antibiotics
Q185 OB/GYN
How are uncomplicated perineal lacerations
(e.g., no fever, purulence) managed (e.g.,
perineal pain, particularly with voiding)?

A) NSAID's and sitz baths


B) Antibiotics
Q186 OB/GYN
If pregnancy related nausea and vomiting do not
subside even after dietary modifications (first-line;
e.g., small frequent meals, bland food), what is the
next step?

A) Benazepril
B) Vitamin B6 and doxylamine succinate (H1 blocker)
Q186 OB/GYN
If pregnancy related nausea and vomiting do not
subside even after dietary modifications (first-line;
e.g., small frequent meals, bland food), what is the
next step?

A) Benazepril
B) Vitamin B6 and doxylamine succinate (H1 blocker)
Q187 OB/GYN
A 35-year-old woman with (no significant medical
history) at 33 weeks gestation is found unresponsive by
her son; she gradually gains consciousness. BP = 138/98.
Frontal lobe edema is seen on CT scan. What is the next
step?

A) Magnesium sulfate
B) CT venography
C) Video EEG
Q187 OB/GYN
A 35-year-old woman with (no significant medical
history) at 33 weeks gestation is found unresponsive by
her son; she gradually gains consciousness. BP = 138/98.
Frontal lobe edema is seen on CT scan. What is the next
step?

A) Magnesium sulfate
B) CT venography
C) Video EEG
Q187 OB/GYN
A 35-year-old woman with (no significant medical
history) at 33 weeks gestation is found unresponsive by
her son; she gradually gains consciousness. BP = 138/98.
Frontal lobe edema is seen on CT scan. What is the next
step?

A) Magnesium sulfate - likely experienced eclampsia


B) CT venography
C) Video EEG
Q188 OB/GYN
A woman who takes amphetamines or cocaine increases
the risk of a fetus with:

A) Fetal malpresentation
B) Vasa previa
C) Gastroschisis
D) Fetal growth restriction
Q188 OB/GYN
A woman who takes amphetamines or cocaine increases
the risk of a fetus with:

A) Fetal malpresentation
B) Vasa previa
C) Gastroschisis
D) Fetal growth restriction
Q189 OB/GYN
A pregnant lady in her first trimester
has gestational thrombocytopenia
(platelets = 110,000). What is the next
step?

A) Abdominal CT scan
B) Routine care
Q189 OB/GYN
A pregnant lady in her first trimester
has gestational thrombocytopenia
(platelets = 110,000). What is the next
step?

A) Abdominal CT scan
B) Routine care
Q189 OB/GYN
A pregnant lady in her first trimester
has gestational thrombocytopenia
(platelets = 110,000). What is the next
step?

A) Abdominal CT scan
B) Routine care - it's a benign condition
that causes isolated thrombocytopenia
Q190 OB/GYN
A pregnant patient comes in at 34 weeks with
breech presentation and a previous classical C
section. What is the next step?

A) C section at 37 weeks
B) Vaginal delivery induction at 40 weeks
C) External cephalic version at 36 weeks
Q190 OB/GYN
A pregnant patient comes in at 34 weeks with
breech presentation and a previous classical C
section. What is the next step?

A) C section at 37 weeks
B) Vaginal delivery induction at 40 weeks
C) External cephalic version at 36 weeks
Q191 OB/GYN
Patients with postpartum urinary retention
should be treated with:

A) Oxybutynin
B) Urethral catheterization
Q191 OB/GYN
Patients with postpartum urinary retention
should be treated with:

A) Oxybutynin
B) Urethral catheterization
Q191 OB/GYN
Patients with postpartum urinary retention
should be treated with:

A) Oxybutynin - for overactive bladder; would


worsen urinary retention
B) Urethral catheterization - intermittent
catheterization
Q192 OB/GYN
Which of the following is a contraindication to
exercise during pregnancy?

A) Cervical insufficiency
B) Preeclampsia
C) Anemia <8 g/L
D) Placenta previa
E) All of the above
Q192 OB/GYN
Which of the following is a contraindication to
exercise during pregnancy?

A) Cervical insufficiency
B) Preeclampsia
C) Anemia <8 g/L
D) Placenta previa
E) All of the above
Q193 OB/GYN
Which activity should ALL pregnant women
avoid?

A) Ice hockey
B) Horseback riding
C) Scuba diving
D) Hot yoga
E) All of the above
Q193 OB/GYN
Which activity should ALL pregnant women
avoid?

A) Ice hockey
B) Horseback riding
C) Scuba diving
D) Hot yoga
E) All of the above
Q194 OB/GYN
A woman at 35 weeks starts developing
frequent painful uterine contractions. Cervix
is 4 cm and 90% effaced. What is the next
step?

A) Nifedipine
B) Expectant management
Q194 OB/GYN
A woman at 35 weeks starts developing
frequent painful uterine contractions. Cervix
is 4 cm and 90% effaced. What is the next
step?

A) Nifedipine
B) Expectant management
Q194 OB/GYN
A woman at 35 weeks starts developing
frequent painful uterine contractions. Cervix is
4 cm and 90% effaced. What is the next step?

A) Nifedipine
B) Expectant management - risk of tocolysis at
34 weeks outweigh neonatal risks of preterm
Q195 OB/GYN
A 71-year-old woman has a firm white vulvar plaque
that it very itchy. What is the next step?

A) Steroids
B) Vulvar biopsy
Q195 OB/GYN
A 71-year-old woman has a firm white vulvar plaque
that it very itchy. What is the next step?

A) Steroids
B) Vulvar biopsy
Q195 OB/GYN
A 71-year-old woman has a firm white vulvar plaque
that it very itchy. What is the next step?

A) Steroids
B) Vulvar biopsy - to evaluate for vulvar cancer
first; if shown to be benign, steroids
Q196 OB/GYN
A woman has PPROM at 33 weeks. What is the next
step?

A) Prophylactic antibiotics and betamethasone


B) Induction of labor
Q196 OB/GYN
A woman has PPROM at 33 weeks. What is the next
step?

A) Prophylactic antibiotics and betamethasone


B) Induction of labor
Q197 OB/GYN
At term gestation, the most likely cause of new-
onset oligohydramnios is _.
Q197 OB/GYN
At term gestation, the most likely cause of new-
onset oligohydramnios is spontaneous rupture of
membranes.
Q198 OB/GYN
A 42-year-old woman has a pathologic nipple
discharge. What is the next step?

A) Ultrasound only
B) Mammogram only
C) Mammogram and ultrasound
Q198 OB/GYN
A 42-year-old woman has a pathologic nipple
discharge. What is the next step?

A) Ultrasound only
B) Mammogram only
C) Mammogram and ultrasound
Q198 OB/GYN
A 42-year-old woman has a pathologic nipple
discharge. What is the next step?

A) Ultrasound only
B) Mammogram only
C) Mammogram and ultrasound - when used in
combination increases detection of intraductal
lesions that cause nipple discharge
Q199 OB/GYN
The initial menstrual cycles in adolescents are
irregular and anovulatory due to:

A) estrogen deficiency
B) excess LH secretion
C) insufficient secretion of gonadotropin releasing
hormone
Q199 OB/GYN
The initial menstrual cycles in adolescents are
irregular and anovulatory due to:

A) estrogen deficiency
B) excess LH secretion
C) insufficient secretion of gonadotropin releasing
hormone
Q200 OB/GYN
A 16-year-old girl complains of intermittent vaginal discharge
for the last several months. The discharge returns after menses
end. Vital signs are normal. The vagina is not erythematous.
There is a white mucoid odorless vaginal discharge, which shows
squamous cells and rare polymorphonuclear leukocytes under
the microscope. What is the diagnosis?

A) Leukorrhea
B) Candida
Q200 OB/GYN
A 16-year-old girl complains of intermittent vaginal discharge
for the last several months. The discharge returns after menses
end. Vital signs are normal. The vagina is not erythematous.
There is a white mucoid odorless vaginal discharge, which shows
squamous cells and rare polymorphonuclear leukocytes under
the microscope. What is the diagnosis?

A) Leukorrhea
B) Candida
Q200 OB/GYN
A) Leukorrhea
-leukorrhea is a white, odorless mucoid cervical discharge that
typically occurs midcycle due to↑estrogen levels prior to
ovulation. Microscopic examination of the discharge reveals no
evidence of inflammation or infection.
B) Candida

You might also like